Sample test questions

Krok 2 Medicine

()

Терапевтичний профiль 2

1. A patient with Morgagni-Adams-Stokes resonance and absence of respiration.What is syndrome has fainted when walking up the the most likely diagnosis in this case? stairs. The skin is pale, the pupils are dilated, tonoclonic spasms are observed, rib cage is A. Spontaneous pneumothorax immobile. Make the diagnosis: B. Hemothorax C. Lobar pneumonia A. Clinical death D. Pulmonary embolism B. Social death E. Acute pleurisy C. Preagony D. Agony 6. A 34-year-old man on the 3rd day E. Biological death of ceftriaxone treatment for acute otitis (daily dosage - 2 grams) developed diarrhea 2. A 32-year-old welder complains of weakness occurring 5-6 times per day. Feces are without and fever. His illness initially presented as mucus or blood admixtures. Temperature is tonsillitis one month earlier. On examination: 36.6oC. Gregersen reaction (occult blood in o temperature - 38.9 C, respirations - 24/min., feces) is negative. Stool culture detected no pulse - 100/min., blood pressure - 100/70 mm pathogenic germs. What is the most likely Hg, hemorrhages on the legs, enlargement of cause of diarrhea in this case? the lymph nodes. Complete blood count shows Hb- 70 g/L, RBC- 2.2·1012/L, WBC- 3.0·109/L A. Antibiotic-associated diarrhea with 32% of blasts, 1% of eosinophiles, 3% of B. Intestinal dysbiosis bands, 36% of segments, 20% of lymphocytes, C. Bacterial overgrowth syndrome and 8% of monocytes, ESR- 47 mm/hour. D. Ulcerative colitis What is the cause of anemia in this case? E. Crohn’s disease (regional enteritis) A. Acute leukemia 7. A 24-year-old patient visited a doctor B. Chronic lympholeukemia complaining of enlargement of his C. Aplastic anema submaxillary lymph nodes. Objectively: D. B12-deficient anemia submaxillary, axillary and inguinal lymph E. Chronic hemolytic anemia nodes are enlarged. Chest X-ray shows: enlarged lymph nodes of mediastinum. Blood 3. After a 5-day-long celebration of his test: erythrocytes - 3.4 · 1012/L, Hb- 100 g/L, daughter’s wedding a 65-year-old patient blood colour index - 0.88, platelets - 190·109/L, ”saw” in his yard many cats, chickens, and rats. · 9 He tried to chase them away, but was scared leucocytes - 7.5 10 /L, eosinophiles - 8%, band off when the animals started scolding him and neutrophiles - 2%, segmented neutrophiles - tried to harm him. What is the likely diagnosis? 67%, lymphocytes - 23%, ESR - 22 mm/hour. What test must be prescribed to verify the A. Delirium tremens cause of lymphadenopathy? B. Senile psychosis C. Schizophrenia A. Open biopsy of the lymph nodes D. Organic brain syndrome B. Abdominal US E. Reactive hallucinosis C. Mediastinum tomography D. Puncture biopsy of the lymph nodes 4. 5 weeks after hypothermia a 22-year- E. Sternal puncture old patient developed fever, weakness, muscle pain, inability to move independently. 8. A 37-year-old woman complains of Objectively: tenderness, induration of headaches, nausea, vomiting, spasms. The shoulder and shin muscles, restricted active onset of the disease occurred the day before due to her overexposure to cold. Objectively: movements, erythema on the anterior surface o of the chest. There is a periorbital edema with fever up to 40 C; somnolence; rigid ; heliotropic erythema. Gottron’s sign is present. Kernig’s symptom is positive on the both What investigation is required for the diagnosis sides; general hyperesthesia. Blood test: verification? leucocytosis, increased ESR. Cerebrospinal fluid is turbid, yellow-tinted. What changes of A. Muscle biopsy the cerebrospinal fluid are most likely? B. Aminotransferase activity C. Pneumoarthrography A. Neutrophilic pleocytosis D. ASO titer B. Lymphocytic pleocytosis E. Rheumatoid factor C. Blood in the cerebrospinal fluid D. Xanthochromia in the cerebrospinal fluid 5. During physical exertion a man suddenly E. Albuminocytological dissociation developed acute chest pain on the right and dyspnea. Objectively he assumes forced half- 9. A 44-year-old patient complains of difficult sitting position in the bed, presents with diffuse urination, sensation of incomplete urinary cyanosis, resting tachypnea of 38/min., the bladder emptying. Sonographic examination right side of the is enlarged and does of the urinary bladder near the urethra not participate in the respiratory process; entrance revealed an oval well-defined percussion on the right reveals tympanic hyperechogenic formation 2x3 cm large that was changing its position during the Терапевтичний профiль 3 examination. What conclusion can be made? A. Progressive angina pectoris A. Stone B. First-time angina pectoris B. Malignant tumour of the urinary bladder C. Stable NYHA functional class II angina C. Urinary bladder polyp pectoris D. Prostate adenoma D. Variant angina pectoris E. Primary ureter tumour E. Acute cardiac infarction 10. A 25-year-old woman complains of fatigue, 14. During appointment with the doctor dizziness, hemorrhagic rashes on the skin. She a man complains of painful itching rashes has been presenting with these signs for a that appeared on his skin under the beard month. Blood test: erythrocytes - 1.0 · 1012/L, and moustache one year ago, with frequent Hb- 37 g/L, colour index - 1.1, leukocytes - exacerbations occurring throughout the year. 1.2 · 109/L, platelets - 42 · 109/L. What analysis Objectively the skin of the facial hair growth would be the most advisable for diagnosis- areas is bluish-purple, thickened, with pustules, making in this case? erosions, and scabs covering its moist surface. The fistulous tracts are surrounded by the A. Sternal puncture (bone marrow biopsy) area of loose pink-red granulation and B. Splenic biopsy discharge pus. The skin resembles mulberry C. Liver biopsy in appearance. Make the diagnosis: D. Coagulation studies E. US of the gastrointestinal tract A. Sycosis B. Acne rosacea 11. A 35-year-old man complains of rapidly C. Lupus erythematosus incresing fatigue, palpitations, ”visual snow”, D. Deep trichophytosis dizziness. He has a history of peptic ulcer of the E. Tuberculous lupus stomach. Objectively the skin is pale. Vesicular respiration is observed in the lungs. Systolic 15. A 60-year-old man presents with ischemic murmur is detected over the cardiac apex, heart disease and heart failure of the IV heart rate is 100/min., BP is 100/70 mm Hg. class according to NYHA (New York Heart The is slightly tender on palpation. Association) that manifests as dyspnea at Blood test: erythrocytes - 3.2 · 1012/L, Нb- 100 rest. There are moist crackles in the patient’s g/L, color index - 0.94. What type of anemia is lungs. Liver +4 cm, lower limbs are swollen. the most likely present in this case? Ejection fraction is 25%. What sign is the most indicative when determining functional class A. Posthemorrhagic anemia of heart failure according to NYHA? B. Sideroblastic anemia Degree of dyspnea C. Iron-deficiency anemia A. D. Hemolytic anemia B. Moist crackles in the lungs Swollen lower limbs E. Hypoplastic anemia C. D. Decrease of ejection fraction 12. A 35-year-old patient has been suffering E. Extent of liver enlargement from an illness for 3 days. 5 days ago he returned from a trip to Africa. The onset of 16. A patient suffering from infiltrative disease was accompanied by fever up to 40oC, pulmonary tuberculosis was prescribed chills, acute headache, myalgia. In the axillary streptomycin, rifampicin, isoniazid, pyrazinamide, region the lymph node enlarged up to 3x6 cm vitamin C. One month after the beginning of can be palpated. The lymph node is dense, the treatment the patient started complaining intensely painful, slightly mobile, without clear of reduced hearing and tinnitus. What drug has margins; the skin over the node is hyperemic suchasideeffect? and tight. Tachycardia is present. Make the A. Streptomycin preliminary diagnosis: B. Isoniazid A. Plague C. Rifampicin Pyrazinamide B. Sepsis D. Vitamin C C. Tu l a r e m i a E. D. Lymphadenitis 17. A 39-year-old man complains of morning E. Anthrax headaches, appetite loss, nausea, morning vomiting, periodic nasal hemorrhages. The 13. A 52-year-old patient, who has been suffering from angina pectoris, for 2 weeks patient had a case of acute glomerulonephritis has increasingly frequent pain attacks in the at the age of 15. Examination revealed rise area behind his sternum and his need for of arterial pressure up to 220/130 mm Hg, nitroglycerine has increased. Objectively: the skin hemorrhages on his and legs, condition is of moderate severity. The skin is pallor of skin and mucous membranes. What pale. Heart sounds are weakened, rhythmic. biochemical parameter is the most important Heart rate is 84/min. ECG shows no signs of for making diagnosis in this case? focal myocardial injury. What is the most likely diagnosis? Терапевтичний профiль 4

A. Blood creatinine center. What disease is most likely in the given B. Blood bilirubin case? C. Blood sodium D. Uric acid A. Pneumonia complicated by an abscess E. Fibrinogen B. Infiltrative tuberculosis C. Peripheral pulmonary cancer 18. The doctor has an appointment with a D. Cystic echinococcosis patient, who 2 days ago developed severe E. Pulmonary cyst chest pain on the left, general weakness, high temperature, and headache. Objectively along 22. A 16-year-old adolescent living in a rural the 4th and 5th intercostal nerves on the left area has been bitten in the shin by a stray dog. the skin is hyperemic and there are tight The wound is superficial. Regular vaccination clusters of small vesicles filled with clear serous against tetanus was received 3 months ago. content. What is the most likely diagnosis? What treatment tactics would be the most advisable in this case? A. Herpes zoster B. Herpes simplex A. Antirabies vaccination C. Streptococcal impetigo B. Antirabies immunoglobulin D. Pemphigus C. Tetanus toxoid adsorbed E. Dermatitis herpetiformis (Duhring’s disease) D. Antitetanus serum E. Antitetanus immunoglobulin 19. For a week a 42-year-old patient has been suffering from fever attacks followed 23. A 48-year-old patient was found to have by high temperature, which occur every 48 diffuse enlargement of the thyroid gland, hours. Body temperature raises up to 40oC and exophthalmia, weight loss of 4 kg in 2 months, decreases in 3-4 hours with excessive sweating. sweating. Objectively: HR- 105/min, BP- The patient presents with loss of appetite 140/70 mm Hg. Defecation act is normal. What and general fatigue. The skin is pale and kind of therapy is recommended in this case? sallow. The liver and spleen are enlarged and dense on palpation. What method of diagnosis A. Mercazolil verification would be most efficient? B. Radioactive iodine C. Propranolol A. Microscopy of blood smear and thick blood D. Lugol’s solution film E. Thyroxine B. Complete blood count C. Bacteriological analysis 24. A 26-year-old woman is suspected to D. Immune-enzyme assay suffer from systemic lupus erythematosus with E. Microscopy of hanging blood drop systemic lesions of skin, vessels, joints, serous tunics, and heart tissue that developed after 20. A 28-year-old patient is a drug addict. He photosensitization. The following is detected has been sick for a year, when noticed general in blood analysis: LE cells, antibodies to native weakness, increased sweating, and weight loss. ds-DNA, isolated anti-centromere antibodies, He often had cases of respiratory diseases. rheumatoid factor is 1:100, Wassermann Within the last 2 days he demonstrates reaction is positive, circulating immune intermittent fever with profuse night sweating, complex is 120 units. What immunological increased general weakness, developed indicators are considered to be specific to this diarrhea with mucus and blood admixtures. On disease? examination: polylymphadenopathy, herpetic rashes in the oral cavity; on abdominal A. ds-DNA antibodies palpation: the liver and spleen are enlarged. B. Rheumatoid factor What is the most likely diagnosis? C. Anti-centromere antibodies D. Immunoglobulin A A. HIV-infection E. Increased circulating immune complex B. Herpetic stomatitis 25. A woman came to a doctor with complaints C. Chronic lymphatic leukemia o D. Colon cancer of increased body temperature up to 37,8 C E. Chronic sepsis and moderately sore throat for the last 3 days. Objectively: mandibular lymph nodes 21. A 40-year-old patient has acute onset are enlarged up to 3 cm. Palatine tonsils are of disease caused by overexposure to cold. hypertrophied, with gray coating that spreads Temperature has increased up to 39oC. to the uvula and anterior pillars of the fauces. Foul-smelling sputum is expectorated during What is the most likely diagnosis? coughig. Various moist crackles can be auscultated above the 3rd segment on the A. Oropharyngeal diphtheria right. Blood test: leukocytes - 15, 0 · 109/l, stab B. Infectious mononucleosis neutrophils - 12%, ESR- 52 mm/hour. On X- C. Pseudomembranous (Vincent’s) tonsillitis ray: in the 3rd segment on the right there is a D. Agranulocytosis focus of shadow 3 cm in diameter, low density, E. Oropharyngeal candidiasis with fuzzy smooth margins and a clearing in its 26. A group of 5 had been resting in a forest, Терапевтичний профiль 5 they were drinking alcohol and eating canned the knee and elbow joints is observed. Pulse mushrooms and cured fish. The next day two of is 106/min., rhythmic. Blood pressure is 90/60 them were hospitalized with disturbed vision, mm Hg. Cardiac borders are unchanged, heart swallowing and respiration; the third one sounds are weakened, at the cardiac apex there presented with acute general weakness and is soft systolic murmur. What factor would dry mouth. The remaining two were healthy. be the most indicative of the likely disease A tick was detected on the skin of one of etiology? the healthy group members. What is the most likely diagnosis? A. Anti-streptolysin O B. C-reactive protein A. Botulism C. Creatine kinase B. Tick-borne encephalitis D. Rheumatoid factor C. Alcohol poisoning E. Seromucoid D. Mushroom poisoning E. Lyme borreliosis 31. A 45-year-old man complains of cough fits and tickling in his nasopharynx. He had 27. A 23-year-old man complains of facial been staying for 10 days in the polluted area edema, headache, dizziness, low urinary created by the Chornobyl nuclear power plant output, urine discoloration (dark red). These accident. Rhinoscopy shows signs of severe complaints arose after the patient had had a nasopharynx irritation. What radionuclide is case of acute tonsillitis. On examination there the cause of this irritation? are facial edema, the skin is pale, temperature is 37.4oC; heart rate is 86/min., blood pressure A. Radioactive iodine is 170/110 mm Hg. Heart sounds are muffled, B. Radioactive cesium the II heart sound is accentuated over the C. Radioactive strontium aorta. What etiological factor is the most likely D. Radioactive plutonium in this case? E. Radioactive cobalt A. Beta-hemolytic streptococcus 32. A 20-year-old patient complains of severe B. Staphylococcus aureus headache, double vision, weakness, fever, irritability. Objectively: body temperature is C. Streptococcus viridans o D. Streptococcus pyogenes 38.1 C, the patient is reluctant to contact, E. Staphylococcus saprophyticus sensitive to stimuli. There are ptosis of the left eyelid, exotropia, anisocoria S>D, pronounced 28. A 65-year-old man was diagnosed with meningeal syndrome. On lumbar puncture the B12-deficient anemia and the treatment was cerebrospinal fluid flowed out under a pressure prescribed. A week later control blood test was of 300 mm Hg, the fluid is clear, slightly performed. What would be the early indicator opalescent. 24 hours later there appeared of the therapy effectiveness? fibrin film. Protein - 1.4 g/L, lymphocytes - 600/3 per mm3,sugar-0.3mmol/L.Whatis A. Increased number of reticulocytes the provisional diagnosis? B. Increased hemoglobin level C. Megaloblastic hematopoiesis A. Tuberculous meningitis D. Normoblastic hematopoiesis B. Meningococcal meningitis E. Increased erythrocyte number C. Lymphocytic (Armstrong’s) meningitis D. Syphilitic meningitis 29. A 35-year-old woman complains of heart E. Mumps meningitis pain (”aching and drilling”) occurring mainly in the morning in autumn and spring and 33. A 32-year-old woman complains of irradiating to the neck, back and ; increasing spastic pains in her lower abdomen rapid heartbeat; low vitality. Occurrence of that occur after emotional stress. Bowel this condition is not associated with physical movements are intermittent: 2-3 defecations activity. In the evening, the patient’s condition after waking in the morning alternate with improves. Study of somatic and neurological constipations that last for 1-2 days. Objectively status and ECG reveal no pathology. What body mass is retained, palpation of the pathology is most likely to have caused these sigmoid colon is moderately painful. Hb- 130 clinical presentations? g/L, leukocytes - 5.2 g/L, ESR- 9 mm/hour. Rectoromanoscopy is painful due to spastic A. Somatization depression condition of the intestine, intestinal mucosa B. Resting stenocardia is without changes. Intestinal lumen contains C. Pseudoneurotic schizophrenia large amounts of mucus. What is the most D. Neurocirculatory asthenia likely diagnosis in this case? E. Hypochondriacal depression A. Irritable bowel syndrome 30. An 18-year-old young man complains of B. Crohn’s disease (regional enteritis) pain in his knee and elbow joints and body o C. Nonspecific ulcerative colitis temperature up to 39.5 C. One week and a half D. Acute mesenteric ischemia earlier developed sore throat. On examination E. Malabsorption syndrome his body temperature is 38.5oC. Swelling of Терапевтичний профiль 6

34. A 37-year-old worker during a fire ended fatigue. He has been presenting with these up in the area of high CO concentration. He symptoms for 3 months. He has been smoking was delivered to a hospital in unconscious since early adolescence. Objectively to is state. Objectively: the skin of his 37.4oC, respirations are 26/min., pulse is and hands is crimson. Respiration rate is 82/min., rhythmic, blood pressure is 130/85 mm 20/min. ECG: alterations specific for hypoxic Hg. The right side of the thorax lags behind myocardium. Hourly diuresis is 40 ml. Blood in the respiratory process, dull percussion test: erythrocytes - 4.5 · 1012/L, Нb- 136 sound and acute decrease of breathing g/L, color index - 0.9, ESR- 3 mm/hour, activity are observed there. X-ray shows carboxyhemoglobin - 5%. What criterion homogeneous shadow of the lung field on the allows determining the severity of the patient’s right with mediastinum displacement towards condition? the affected side. What is the most likely diagnosis? A. Carboxyhemoglobin concentration B. Respiratory disorders A. Central lung cancer C. ECG results B. Exudative pleuritis D. Extent of trophic disorders C. Pleuropneumonia E. Development of chronic renal failure D. Pulmonary tuberculosis E. Multiple bronchiectasis 35. A patient with suspected pheochromocytoma has normal blood pressure in the periods 39. A 42-year-old woman complains of severe between the atacks and a tendency pulsing headache in the frontoparietal area, towards tachycardia. Urine test revealed vertigo, palpitations. She has been suffering no pathologies. It was decided to use from hypertension for 3 years. Significant a provocative test with histamine. What increase of BP occurs 2-3 times per month medication should be prepared to provide and lasts for 3-8 hours. The left ventricle is emergency care in case of positive test result? enlarged, heart sounds are clear, heart rate - 105/min., BP- 225/115 mm Hg. ECG: signs A. Phentolamine of left ventricular hypertrophy. What drug B. Pipolphen would be the most effective for termination C. Nifedipine of cerebral crisis attack? D. Mesatonum E. Prednisolone A. Labetalol B. Hydrochlorothiazide (Hypothiazide) 36. A 46-year-old woman complains of C. Captopril severe pain attacks in the right lumbar D. Losartan area, which irradiate to the lower abdomen, E. Clonidine (Clophelin) and nausea. This kind of pain attacks has never been detected in the patient before. 40. A 45-year-old patient complains of pain Plain abdominal X-ray reveals no pathologic in the epigastric region, left subcostal area, shadows. Ultrasound detects a hyperechogenic abdominal distension, diarrhea, loss of weight. mass 1.5 cm in diameter, which reflects sound He has been suffering from this condition for wave, in the enlarged right renal . What 5 years. Objectively: the tongue is moist with diagnosis is the most likely? white coating near the root; deep palpation of abdomen reveals slight pain in the epigastric A. Renal calculus region and Мауо-Robson’s point. Liver is B. Benign renal tumor painless and protrudes by 1 cm from the costal C. Renal cyst arch. Spleen cannot be palpated. What disease D. Renal tuberculosis can be primarily suspected? E. Malignant renal tumor A. Chronic pancreatitis 37. A 60-year-old woman complains of B. Atrophic gastritis unbearable pain in her right subcostal area. C. Peptic stomach ulcer She has a history of acute pancreatitis. D. Chronic cholecystitis Temperature is 38.2oC. Objectively her sclera E. Chronic enteritis are icteric. There are no signs of peritoneal irritation. Ortner’s and Gubergrits’ symptoms 41. For 4 days a 35-year-old man has been are positive. Urine diastase is 320 U/L. What is treated in the resuscitation unit for acute renal the most likely diagnosis? failure caused by compartment syndrome. The patient is disoriented. ECG shows high A. Exacerbation of chronic pancreatitis T waves and right ventricular extrasystoles. B. Acute cholangitis His central venous pressure is 159 mmH2O; C. Chronic cholecystitis for the last 3 hours auscultation has been D. Acute cholecystitis detecting isolated moist crackles in the lungs. E. Pancreatic cancer Respirations are 32/min. Blood test: residual nitrogen - 62 mmol/L, K+ - 7.1 mmol/L, Cl− - 38. A 64-year-old man complains of cough with Na+ expectoration consisting of blood-streaked 78 mmol/L, - 120 mmol/L, Ht- 0.32 L/L, mucus, dyspnea, low grade fever and general Hb- 100 g/L, blood creatinine - 0.9 mmol/L. In this case the most advisable would be to Терапевтичний профiль 7 perform: condition for 2 years. Objectively she presents with pale skin, subicteric sclera, and bright- A. Hemodialysis red fissured tongue. Lymph nodes are not B. Plasmasorption enlarged. Pulse - 100/min. BP- 105/70 mm Hg. C. Hemosorption Liver +3 cm, the spleen cannot be palpated. D. Plasmafiltration Blood test: erythrocytes - 1.2 · 1012/L, Нb- 56 E. Ultrafiltration g/L, color index - 1.4, macrocytes, leukocytes 9 42. For three years a 31-year-old woman - 2, 5 · 10 /L, eosinophils - 1%, juvenile - 1%, has been complaining of pain and swelling metamyelocytes - 1%, band neutrophils - 8%, of her radiocarpal and metacarpophalangeal segmented neutrophils - 47%, lymphocytes - articulations, their reduced mobility in the 38%, monocytes - 4%, reticulocytes - 0.1%, morning, which persisted up to 1,5 hours. platelets - 100 · 109/L, ESR- 30 mm/hour, Two weeks ago she developed pain, swelling, indirect bilirubin - 26 mmol/L. What changes and reddening of her knee joints, her body can be expected in the bone marrow puncture temperature increased up to 37,5oC.The material? treatment was belated. Examination of the internal organs revealed no pathologic A. Prevalence of megaloblasts alterations. Diagnosis of rheumatoid arthritis B. Increased number of sideroblasts was made. What alterations are the most likely C. Erythroid hyperplasia to be visible on the arthrogram? D. Presence of blast cells E. Prevalence of lymphoid tissue A. Joint space narrowing, usuration B. Joint space narrowing, subchondral 46. A 58-year-old woman complains of osteosclerosis spontaneous bruises, weakness, bleeding gums, C. Cysts in the subchondral bone dizziness. Objectively: the mucous membranes D. Numerous marginal osteophytes and skin are pale with numerous hemorrhages E. Epiphyseal osteolysis of various time of origin. Lymph nodes are not enlarged. Ps- 100/min, BP- 110/70 mm Hg. 43. A 55-year-old woman, a cook, complains There are no alterations of internal organs. of pain in her right knee joint that has been Blood test results: RBC- 3, 0·1012/l, Нb- 92 g/l, troubling her for a month and intensifies in color index - 0,9, anisocytosis, poikilocytosis, the evening. Objectively she is overweight, the WBC- 10 · 109/l, eosinophils - 2%, stab knee joint is swollen, creaks during movement, neutrophils - 12%, segmented neutrophils - palpation reveals localized pain. The 1st 68%, lymphocytes - 11%, monocytes - 7%, metatarsophalangeal articulation is deformed ESR- 12 mm/h. What laboratory test should be on the both feet. No changes in blood and performed additionally to make the diagnosis? urine are detected. What should be visible on the patient’s X-ray? A. Platelets B. Reticulocytes A. Joint space narrowing, marginal osteophytes C. Clotting time B. Osteoporosis, joint space narrowing, singular D. Osmotic resistance of erythrocytes usurations E. Fibrinogen C. Joint space narrowing, multiple usurations, subluxations 47. A 45-year-old woman complains of D. Epiphyseal erosions, bony ankylosis paroxysmal intolerable facial pain on the left E. Joint space narrowing, round bone defects with attacks that last for 1-2 minutes. Attacks are provoked by chewing. The disease onset 44. A 52-year-old woman has been suffering was two month ago after overexposure to for 2 years from dull, occasionally exacerbating cold. Objectively: pain at the exit points of pain in her right subcostal area, occurring after the trigeminal nerve on the left. Touching near eating high-fat foods, bitter taste in her mouth the wing of nose on the left induces new pain in the morning, constipations, and flatulence. attack with tonic spasm of the facial muscles. Objectively she has excess weight, her body o What is the most likely diagnosis among those temperature is 36.9 C; there is a coating listed? on the root of her tongue; the abdomen is moderately distended and painful in the area A. Trigeminal neuralgia of gallbladder projection. What examination B. Glossopharyngeal neuralgia would be the most helpful for diagnosis- C. Temporomandibular joint arthritis making? D. Facial migraine E. Maxillary sinusitis A. Ultrasound B. Duodenal intubation 48. A 28-year-old man complains of skin C. Cholecystography rash and itching on the both of his D. Duodenoscopy hands. The condition persists for 1,5 years. E. Liver scanning The exacerbation of his condition he ascribes to the occupational contact with 45. A 57-year-old woman complains of formaldehyde resins. Objectively: lesion foci weakness, dyspnea, loss of appetite, and are symmetrically localized on both hands. liquid feces. She has been suffering from this Терапевтичний профiль 8

Against the background of erythema with A. Renal amyloidosis blurred margins there are papulae, vesicles, B. Chronic glomerulonephritis erosions, crusts, and scales. What is the most C. Acute glomerulonephritis likely pathology? D. Heart failure E. Chronic pyelonephritis A. Occupational eczema B. Idiopathic eczema 52. A 23-year-old patient had taken 1 g of C. Allergic dermatitis aspirin to treat acute respiratory infection. D. Simple contact dermatitis After that he developed an asthmatic fit E. Erythema multiforme with labored expiration that was arrested by introduction of aminophylline. The patient’s 49. A 25-year-old patient was delivered to medical history is not burdened with allergies. an infectious diseases unit on the 3rd day The patient has undergone two surgeries for of illness with complaints of headache, pain nasal polyposis in the past. What diagnosis is in lumbar spine and gastrocnemius muscles, most likely? high fever, chill. Objectively: condition of moderate severity. Scleras are icteric. Pharynx A. Aspirin-induced asthma is hyperemic. Tongue is dry with dry brown B. Atopic bronchial asthma coating. Abdomen is distended. Liver is C. Infectious allergic bronchial asthma enlarged by 2 cm. Spleen is not enlarged. D. Exercise-induced asthma Palpation of muscles, especially gastrocnemius E. Symptomatic bronchospasm muscles, is painful. Urine is dark in colour. Stool is normal in colour. The most likely 53. A 36-year-old man has been complaining diagnosis is: of marked weakness, low appetite, and nausea for the last year, year and a half. Within the A. Leptospirosis last year he has lost 10 kg of body mass. B. Viral hepatitis type A The skin is darkened, especially on his face, C. Malaria neck, and arms. Skin folds and nipples are D. Infectious mononucleosis hyperpigmented, there are pigment spots on E. Yersiniosis the patient’s inner thighs. Pulse is 60/min., blood pressure is 80/50 mm Hg. What is the 50. A 28-year-old woman complains of provisional diagnosis? skin hemorrhages after minor traumas and spontaneous appearance of hemorrhages on A. Adrenal insufficiency the front of her torso and extremities. On B. Diabetes mellitus examination: the skin is variegated (old and C. Chronic gastritis new hemorrhages), bleeding gums. Blood D. Hemochromatosis platelets - 20 · 109/L; in the bone marrow there E. Cholestatic hepatitis is increased number of megakaryocytes and 54. A 45-year-old man developed constricting no platelet production. Treatment with steroid retrosternal pain that occurs during walks hormones was effective. What is the likely at the distance of 200 m. Objectively heart diagnosis? rate is 80/min., BP is 160/90 mm Hg. During cardiopulmonary exercise test at 50 W there is A. Idiopathic thrombocytopenic purpura Hemophilia a depression of S-T segment by 3 mm below B. the isoline in V3-V4. What is the provisional C. Rendu-Osler-Weber disease (Hereditary diagnosis? hemorrhagic telangiectasia) D. Disseminated intravascular coagulation A. Exertional angina pectoris, functional class E. Acute vascular purpura III B. Exertional angina pectoris, functional class 51. A 46-year-old man notes swollen legs, IV weakness, sensation of fullness and heaviness C. Exertional angina pectoris, functional class in the right subcostal area; it is the first II occurrence of these signs in the patient. The D. Somatoform autonomic dysfunction, patient has 20-year-long history of rheumatoid hypertension type arthritis. The liver and spleen are enlarged E. Alcoholic myocardiodystrophy and dense. Blood creatinine - 0,23 mmol/l, proteinemia - 68 g/l, cholesterol - 4,2 mmol/l, 55. For the last 15 years a 48-year-old urine specific gravity - 1012, proteinuria - patient has been working at the factory 3,3 g/l, isolated wax-like cylinders, leached producing synthetic resins. Lately he has been erythrocytes in the vision field, leukocytes - complaining of significant general fatigue, 5-6 in the vision field. What is the most likely headaches, frequent urination (predominantly complication? during the day), red color of urine. What complication of benzene nitrocompounds poisoning can be suspected? Терапевтичний профiль 9

A. Malignant tumor of the urinary bladder 165 cm. The patient’s skin is clear, with even B. Chronic cystitis distribution of subcutaneous fat; the thyroid C. Chronic prostatitis gland is not enlarged; there are no menstrual D. Acute glomerulonephritis cycle disturbances. What obesity prevention E. Chronic pyelonephritis methods would be the most advisable in this case? 56. A 44-year-old patient with postinfarction cardiosclerosis presents with frequent heart A. Dietary treatment, graduated exercise rate disorders and lower extremity edema. B. Intensive training regimen Objectively: Ps- 95/min., irregular, 10-12 C. Gastroplasty or gastrojejunal shunt extrasystoles per minute. BP- 135/90 mm Hg. D. Inhibitors of gastrointestinal lipases The 1st heart sound at the apex is weakened. E. Anorectic drugs Pulmonary respiration is rough. The liver is enlarged +2 cm. ECG: irregular sinus 60. A 57-year-old woman complains of rhythm, heart rate - 95/min., frequent polytopic a sensation of esophageal compression, ventricular extrasystoles. What antiarrhythmic palpitations, difficult breathing when eating drug is advisable in this case for treatment and solid food, occasional vomiting with a full prevention of extrasystole? mouth, ”wet pillow” sign at night for the last 6 months. Objectively: body tempearture A. Amiodarone -39oC, height - 168 cm, weight - 72 kg, B. Lidocaine Ps- 76/min, BP- 120/80 mm Hg. X-ray study C. Mexiletine revealed considerable dilation of esophagus D. Quinidine and its constriction in the cardial part. What E. Novocainamide (Procainamide) pathology is the most likely to have caused dysphagia in this patient? 57. A 60-year-old woman started feeling weakness, vertigo, rapid fatigability during the A. Achalasia cardiae last year. Recently she has developed dyspnea B. Primary esophageal spasm and paresthesia observed. Objectively: skin C. Hiatal hernia and mucous membranes are pale and icteric. D. Esophageal carcinoma Lingual papillae are smoothed out. Liver and E. Reflux esophagitis spleen are at the edge of costal arch. Blood test: Hb- 70 g/L, erythrocytes - 1.7 · 1012/L, 61. A 43-year-old woman complains of pain in blood color index - 1.2, macrocytes. What drug the lumbar area, which irradiates to her left leg can be prescribed on pathogenetic grounds? and aggravates on movement, and sensation of numbness in this leg. Objectively palpation A. Vitamin B12 of her shin and thigh is painful, there are B. Vitamin B6 painful stretch symptoms of on the left and C. Ascorbic acid gastrocnemius cramps. There is no sensory loss D. Iron preparations or weakening of reflex responses. Make the E. Vitamin B1 diagnosis: 58. After excessive consumption of fatty food A. Vertebrogenous lumbar ischialgia on the left a 60-year-old woman suddenly developed B. Vertebrogenous radicular syndrome of L5-S1 pain in her right subcostal area, nausea, on the left bile vomiting, sharp bitter taste in her C. Left-sided coxitis mouth. In 2 days she developed jaundice, D. Endarteritis of the lower extremities her urine darkened. Objectively: sclera and E. Spinal stroke skin are icteric, the abdomen is distended, the liver is enlarged by 3 cm, soft and painful 62. A patient with chronic pancreatitis on palpation, Ortner’s, Murphy’s, Kehr’s, complains of diarrhea occurring up to 5 times Zakharyin’s, Mayo-Robson’s signs are positive. per day (no blood traces), loss of body weight, What diagnostic technique should be used in abdominal distention, dryness of skin, loss the first place to confirm the diagnosis? of hair, thirst, bleeding gums, convulsions. Complete blood count: leukocytes - 5.8 · 109/L; A. Ultrasound of the gallbladder and bile duct Hb- 86 g/L; ESR- 15 mm/g; Blood protein B. Fibrogastroduodenoscopy test: total protein - 48 g/L; albumins - 28 g/L. C. Abdominal X-ray What indicators of coprological analysis would D. Radionuclide scanning of the liver and accompany this syndrom? gallbladder E. Laparoscopy A. Steatorrhea, creatorrhea B. Large amount of mucus, amylorrhea 59. A 30-year-old woman complains of C. Large amount of starch grains and cellulose increased body weight and problems with D. Gas bubbles, acid reaction physical exertion. Her parents are of increased E. Large numbers of iodinophilous microbes body weight as well; typical meals in their family are high in fats and carbohydrates. 63. A 35-year-old patient complains of Objectively her blood pressure is 135/80 mm heartburn, gasseous and sour eructation, Hg, pulse is 89/min., weight is 87 kg, height is burning constricting pain behind the sternum Терапевтичний профiль 10 and along the esophagus, developing when A. Neuroleptics bowing the torso to the front. No previous B. Antidepressants examination; the patient takes almagel at C. Tranquilizers his own discretion, after which he notes D. Nootropics improvement of his general state. Make the E. Mood stabilizers provisional diagnosis: 68. A young woman suffering from seborrhea A. Gastroesophageal reflux disease oleosa has numerous light-brown and white B. Functional dyspepsia spots on the skin of her torso and shoulders. C. Cardiospasm The spots have clear margins, branny D. Ulcer disease of the stomach desquamation, no itching. What provisional E. Ulcer disease of the duodenum diagnosis can be made? 64. A 34-year-old man is being treated for A. Pityriasis versicolor schizophrenia exacerbation in a psychiatric B. Torso dermatophytosis unit. Objectively: the patient remains in bed, C. Seborrheic dermatitis is sluggishly mobile, unresponsive, does not D. Pityriasis rosea react to questions. His position is unvaried, E. Vitiligo hypomimic, snout reflex and Dupre’s symptom are present, muscles exhibit waxy flexibility. 69. A man came to an urologist with complains He has been remaining in this state for of painful urination, discharge from urethra. approximately a week. Feeding is parenteral. The patient has been suffering from this Determine the neuromotor disturbance: condition for a week. Objectively: hyperemic urinary meatus, edema, purulent discharge. A. Catatonic stupor Microscopy of smears detected gram-negative B. Depressive stupor bacteria. Specify the diagnosis: C. Psychogenic stupor D. Anergic stupor A. Acute gonorrheal urethritis E. Exogenic stupor B. Trichomonas urethritis C. Candidal urethritis 65. After overexposure to cold a 45-year-old D. Chlamydial urethritis woman developed acute pain in her suprapubic E. Chronic gonorrhea and lumbar areas during urination, sharp pains at the end of urination, false urges to urinate. 70. A patient is 28 years old. He has been Urine is turbid with blood streaks. The doctor suffering from mental disorder since he was suspects urinary tract infection. What results 22. His current condition has changed acutely: of laboratory analysis would be the most for 3 days the patient has been refusing to indicative of such infection? leave his home. He claims that there is a ”telepathy” occurring between him and other A. Leukocyturia, gross hematuria people, through which he receives ”thoughts B. Gross hematuria of strangers” and transmits his own thoughts C. Increased blood creatinine and blood urea for everyone to hear. He thinks his thoughts D. Daily proteinuria under 3.0 and actions are manipulated through this E. Daily proteinuria over 3.0 ”telepathy”. Make the preliminary diagnosis: 66. A 35-year-old patient developed an A. Paranoid schizophrenia epileptic attack with tonoclonic spasms that B. Depressive episode lasted for 3 minutes. After the attack the C. Catatonic episode patient fell asleep but in 5 minutes the second D. Organic delirium attack occurred. The first step of emergency E. Acute reaction to stress aid would be to: 71. A 35-year-old woman has gained 20 kg A. Ensure patency of airways weight within a year with the normal diet. B. Take blood from the vein for analysis She complains of chills, sleepiness, shortness C. Introduce diazepam intravenously of breath. The patient’s mother and sister D. Prescribe antiepileptic drugs are corpulent. Objectively: height - 160 cm, E. Administer chloral hydrate via enema weight - 92 kg, BMI- 35,9. Obesity is uniform, there are no striae. The face is amimic. The 67. A 23-year-old woman has been suffering skin is dry. The tongue is thickened. Heart from a mental disease since the age of 18, the sounds are muffled. HR- 56/min, BP- 140/100 course of disease has no remission periods. mm Hg. The patient has been suffering from At a hospital the patient mostly presents amenorrhea for 5 months, has constipations. with non-purposeful foolish excitation: she TSH- 28 mcIU/l (norm is 0,32-5). Craniogram makes stereotypic grimaces, exposes herself, shows no pathology. What is the etiology of publicly masturbates with a loud laughter, obesity? repeates stereotypical abusive shouts. The patient should be prescribed: Терапевтичний профiль 11

A. Hypothyroid Barrel chest is observed, on percussion there B. Hypo-ovarian is vesiculotympanitic (bandbox) resonance, C. Hypothalamic-pituitary sharply weakened vesicular respiration on D. Alimentary and constitutive both sides, moist crepitant crackles in the E. Hypercorticoid lower segments of the lungs. Heart sounds are weakened, the II heart sound is accentuated 72. A patient complains of painless ”sores” on his penis and inguinal lymph nodes over the pulmonary artery. The liver is +3 cm. What complicated the clinical course of COPD enlargement. Synthomycin emulsion that the patient have been applying to the ”sores” was in this patient? ineffective. Objectively: on the inner leaf of A. Chronic pulmonary heart the foreskin there are three closely situated B. Pulmonary embolism rounded erosions, 0,5 cm in diameter, with C. Acute left ventricular failure dense infiltration that can be palpated at their D. Diffuse pneumosclerosis bases. Make the preliminary diagnosis: E. Community-acquired pneumonia A. Primary syphilis 76. A 72-year-old man with pnaumonia B. Herpes simplex (Herpes pro genitalis) complains of marked dyspnea, chest pain, C. Candidiasis of the inner leaf of the foreskin severe cough with expectoration, to is 39.5- D. Shingles 40oC, no urination for a whole day.Objectively E. Erythema multiforme the patient is conscious. Respiratory rate is 36/min. Over the right lower pulmonary lobe 73. A 19-year-old student was urgently percussion sound is dull; on auscultation there hospitalized due to marked dyspnea and chest is bronchial respiration and numerous moist pain on the left. Her body temperature is o crackles. Blood pressure is 80/60 mm Hg. Heart 38.8 C. She has been presenting with these rate is 120/min. Heart sounds are muffled, signs for 3 days. Respiratory rate is 42/min., there is tachycardia. What tactics should the shallow. Percussion sound is dull to the left family doctor choose in the management of from the center of the scapula, no respiration this patient? can be auscultated. The left heart border is displaced outwards by 3 cm. Embryocardia A. Hospitalization into intensive care unit and heart rate of 110/min are observed. B. Outpatient treatment Palpation of the right subcostal area is painful. C. Treatment in the day patient facility What urgent measures should be taken in this D. Hospitalization into pulmonology unit case? E. Hospitalization into neurology unit A. Urgent thoracocentesis 77. After the contact with chemicals a B. Prescription of penicillin antibiotics plant worker has suddenly developed C. Administration of furosemide stridor, voice hoarseness, barking cough, D. Administration of cardiac glycosides progressing dyspnea. Objective examination E. Referral into thoracic surgery unit reveals acrocyanosis. What is the provisional diagnosis? 74. A 48-year-old man complains of fatigue, excessive sweating, severe skin itching, A. Laryngeal edema undulant fever, enlarged cervical and B. Laryngeal carcinoma supraclavicular lymph nodes. Objectively: C. PATE paleness of skin and mucosa, cervical lymph D. Pulmonary atelectasis nodes are mobile, dense, elastic, walnut-sized, E. Pneumothorax painless, not attached to the skin. Complete blood count: erythrocytes - 3, 0 · 1012/l, Hb- 78. A 58-year-old man complains of general 100 g/l, leukocytes - 14 · 109/l, eosinophils - weakness, loss of 10 kg of weight within 6%, basophils - 3%, band neutrophils - 11%, 1,5 months, progressive pain in the lumbar segmented neutrophils - 69%, lymphocytes - 7, region, increased blood pressure up to 220/160 280 · 109 mm Hg, low grade fever. Objectively: in the monocytes - 4%, platelets - /l, ESR- 37 right deep palpation reveals mm/hour. What method should be applied to a formation with uneven surface and low verify the diagnosis? mobility; veins of the spermatic cord and A. Lymph node biopsy scrotum are dilated. Blood test results: Hb- 86 B. Sternal puncture g/l, ESR- 44 mm/h. Urine test results: specific C. Muscle biopsy gravity - 1020, protein - 0,99 g/l, RBC cover the D. Chest X-ray whole field of vision, WBC- 4-6 in the field of E. Lumbar puncture vision. What is the provisional diagnosis? 75. A 72-year-old man complains of lower A. Renal tumour extremity edema, sensation of heaviness in B. Urolithiasis the right subcostal area, dyspnea in rest. For C. Acute pyelonephritis over 25 years he has been suffering from D. Acute glomerulonephritis COPD. Objectively: orthopnea, jugular venous E. Nephroptosis distention, diffuse cyanosis, acrocyanosis. 79. Anamnesis of a 30-year-old patient Терапевтичний профiль 12 includes closed thoracic injury. Lately the dyspnea and cardiac pain. He ascribes his patient has been suffering from increasing disease to the case of influenza that he had 2 dyspnea, sensation of heaviness in the right weeks ago. Objectively he leans forward when subcostal area, and heart rate disturbances. sitting. The face is swollen, cyanotic, cervical Objectively: acrocyanosis, bulging cervical veins are swollen. Heart borders are extended veins, ascites, edema of the lower extremities. on the both sides, heart sounds are muffled, Heart auscultation reveals muffled heart heart rate = Ps = 118/min., BP is 90/60 mm Hg. sounds, additional III heart sound is Blood test: ESR is 16 mm/hour. ECG shows detected. Provisional diagnosis of constrictive low voltage. X-ray shows trapezoidal cardiac pericarditis was made. What diagnostic silhouette and signs of pulmonary congestion. technique would NOT confirm the diagnosis? Choose the treatment tactics: A. US of abdomen A. Pericardial puncture (pericardiocenthesis) B. Computer tomography B. Diuretics C. Echocardiography C. Antibiotics D. Magnetic resonance imaging D. Pericardectomy E. Chest X-ray E. Glucocorticosteroids 80. A 72-year-old woman suffers from diabetes 84. A 53-year-old woman complains of weight mellitus type II, concomitant diseases are loss up to 10 kg within the last 2 years, stage II hypertension and stage IIB heart liquid foul-smelling stool two times a day that failure. She takes metformin. Hypertensic poorly washes off the toilet, periodic bouts crisis had occurred the day before, after which of nausea, girdle pain in the upper abdomen. the patient developed extreme weakness, Objectively: pain in Gubergrits zone (on the myalgias, thirst, dry mouth, polyuria. BP is right from navel) and at Mayo-Robson’s point. 140/95 mm Hg, heart rate is 98/min., no edemas Biochemical blood analysis: glucose - 3,2 or smell of acetone detected. What measures mmol/l, bilirubin - 16,5 mcmol/l, crude protein should be taken to prevent development of - 56,4 g/l. Urine diastase/amylase - 426 g/h/l. comatose state in the patient? D-xylose test (oral administration of 25 g of A. Stop metformin, prescribe short-acting d-xylose) after 5 hours reveals 3 g of xylose in insulin urine. The most likely diagnosis is: B. Double the dosage of metformin A. Pancreatitis. Malabsorption syndrome C. Apply hypotonic solution of sodium chloride B. Pseudomembranous colitis D. Additionally prescribe long-acting insulin C. Nonspecific ulcerative colitis E. Prescribe glibenclamide D. Irritable bowel syndrome 81. Survey radiograph of a 52-year-old worker E. Chronic gastritis of an agglomeration plant (28-year-long record 85. A 15-year-old teenager has undergone of service, the concentration of metal dust 3 medical examination in military recruitment is 22-37 mg/m ) shows mildly pronounced center. The following was revealed: interval interstitial fibrosis with diffused contrasting systolic murmur at the cardiac apex, accent of well-defined small nodular shadows. The the II heart sound over the pulmonary artery, patient has no complaints. Pulmonary function tachycardia. What additional examination is not compromised. What is the provisional method will be the most informative for diagnosis? determining diagnosis? A. Siderosis A. Echocardiography B. Silicosis B. Electrocardiography C. Anthraco-silicatosis C. X-ray D. Silicatosis D. Phonocardiography E. Anthracosis E. Rheography 82. A 59-year-old patient suffering from 86. A 25-year-old patient is not married and hypertension was delivered to the hospital has sexual relations with several partners. with complaints of acute headache, nausea, During the last 3 months he noticed small recurrent vomiting. On examination she amount of mucoserous secretions produced presents with acute meningeal symptom from urethra. Subjectively: periodical itching complex. BP is 185/105 mm Hg. What or burning pain in urethra. Two months ago additional examination would you recommend pain in knee join developed. Possibility of to the patient in the first place? trauma or exposure to cold is denied by the patient. During the last week eye discomfort A. Lumbar puncture is noted - lacrimation and itching. What B. Ventriculopuncture provisional diagnosis can be suggested? C. Echoencephalography D. Rheoencephalography E. Electroencephalography 83. A 36-year-old man complains of marked Терапевтичний профiль 13

A. Reactive arthritis color index - 0,92, leukocytes - 7, 4 · 109/l, B. Rheumatoid arthritis platelets - 240 · 109/l, ESR- 11 mm/hour. C. Seasonal pollinosis What measure would most effectively decrease D. Bacterial nonspecific urethral conjunctivitis hemorrhaging in this case? E. URTI with conjunctiva and joints affected A. Cryoprecipitate 87. A 19-year-old woman complains of severe B. Aminocapronic acid pain in the axillary crease. Condition onset C. Native plasma occurred a week ago after her swimming in a D. Direct transfusion of donor blood cold river and epilation. The next day a painful E. Platelet concentrate transfusion ”boil” appeared. The ”boil” was increasing in size every day and became a plum-sized 91. A 28-year-old woman complains of nausea, tumor. Upon examination there are nodular stomachache, pain in her tongue, and liquid conical growths joined together detected, the feces. Three days ago she ate poorly salted skin covering them is bluish-red in color. Some pike caviar. Objectively her skin is pale, the nodules have fistulous openings producing tongue looks ”lacquered” (bald tongue). Pulse thick purulent mass. Body temperature is o is 100/min., with muffled heart sounds and 38, 5 C, general malaise. What is the most systolic murmur over the cardiac apex. Blood likely diagnosis? pressure is 95/50 mm Hg. The liver is enlarged by 3 cm. Hemogram shows anemia, eosinophils A. Hydradenitis - 18%. Oval helminth eggs were detected in B. Carbuncle feces. Make the provisional diagnosis: C. Cutaneous tuberculosis D. Necrotizing ulcerative trichophytosis A. Diphyllobothriasis E. Pyoderma chancriformis B. Trichinosis C. Te n i a s i s 88. A woman complains of weight gain, chills, Taeniarhynchosis fi D. edema, xeroderma, somnolence, dif culties E. Ascaridiasis with focusing. Objectively: height is 165 cm; weight is 90 kg; body proportions are o o 92. A 51-year-old woman complains of female type, t - 35,8 C, heart rate - of headache, trembling, paresthesiae, 58/min., BP- 105/60 mm Hg. Heart sounds palpitations, increased blood pressure up are weakened, bradycardia is observed. Other to 280/160 mm Hg. The day before she internal organs have no alterations. Thyroid experienced exhausting headache, vascular gland cannot be palpated. Milk secretion pulsation, palpitations, asphyxia, stomachache, from mammary glands is observed. Hormone unbearable fear of coming death. The patient test revealed increased levels of thyroid- paled and broke out in cold sweat. In urine stimulating hormone (TSH) and prolactin, and there is increased content of vanillylmandelic decreased level of thyroxine (4). What is the acid. What disease causes such clinical cause of obesity? presentation in the patient? A. Primary hypothyroidism A. Pheochromocytoma B. Secondary hypothyroidism B. Conn’s syndrome (primary C. Prolactinoma hyperaldosteronism) D. Hypopituitarism C. Cushing’s syndrome E. Adiposogenital dystrophy D. Primary hypertension E. Cushing’s disease 89. During hemotransfusion the patient developed nausea, tremor, lumbar and 93. A 26-year-old patient with affective bipolar retrosternal pain. On examination the skin disorder has developed a condition manifested is hyperemic, later developed pallor; the by mood improvement, behavioural and sexual patient presents with hyperhidrosis, labored hyperactivity, verbosity, active body language, respiration, pulse is 110/min., BP is 70/40 mm reduced need for sleep. Which of the following Hg. Urine is black colored. What complication drugs would be most effective in this case? developed in the patient? A. Neuroleptics with sedative effect A. Posttransfusion shock B. Antidepressants with activating effect B. Acute renal failure C. Neuroleptics with activating effect C. Pulmonary embolism D. Tranquilizers D. Anaphylactic shock E. Antidepressants with sedative effect E. Hypotonic crisis 94. A 19-year-old patient complains of dyspnea 90. A 22-year-old man suddenly developed on exertion. He often has bronchitis and extreme weakness, nausea, vomiting with pneumonia. Since childhood the patient traces of blood. The patient is known to presents with cardiac murmur. Auscultation suffer from peptiv ulcer disease of duodenum revealed splitting of the II sound above the and hemophilia A. Objectively: heart rate - pulmonary artery, systolic murmur in the 3rd 102/min., BP- 100/60 mm Hg. Complete blood intercostal space at the left sternal border. count: erythrocytes - 3, 2 · 1012/l, Hb- 98 g/l, ECG detected right bundle branch block. Терапевтичний профiль 14

What is the provisional diagnosis? diagnosis? A. Atrial septal defect A. Nephrotuberculosis B. Open ductus arteriosus B. Right renal cyst C. Aortarctia C. Right renal carcinoma D. Aortic stenosis D. Acute glomerulonephritis E. Mitral insufficiency E. Chronic pyelonephritis 95. A 37-year-old man suffers from attacks 99. A 36-year-old man developed a disease of unconsciousness, dyspnea during physical with acute onset 6 hours ago. The patient exertion, periodical sensations of heart rate presents with pain in the epigastric, ileocecal, disorder. Father of the patient died suddenly and paraumbilical areas, vomiting, weakness, at the age of 45. Objectively: heart rate nausea, and body temperature of 38,5oC. Stool is 90/min., BP is 140/90 mm Hg. On heart is liquid, profuse, frequent, retains fecal nature, US: ejection fraction - 55%, significant foul-smelling, frothy, colored dark green. The myocardium thickening of the left ventricle stomach is moderately distended and painful and interventricular septum. What drug should on palpation. The patient attributes his disease be prescribed for the treatment? to eating raw chicken eggs one day before the clinical signs of the disease appeared. What is A. Bisoprolol the most likely diagnosis? B. Enalapril C. Phenyhydinum (Nifedipine) A. Salmonellosis D. Hydrochlorothiazide B. Shigellosis E. Furosemide C. Typhoid fever D. Cholera 96. A 60-year-old man complains of discomfort E. Food toxicoinfection when swallowing solid food, which he has been observing for a month. He changed his 100. After the celebratory feast that took diet to semiliquid food products. At first the place the day before, a 35-year-old man was discomfort had abated but later it renewed hospitalized with complaints of marked pain despite the change in the diet. The patient within the I metatarsophalangeal articulation developed gaseous eructation and hoarse on the right, which developed late in the voice. What examination should be performed night, and impaired walking. Objectively: the to clarify the diagnosis? metatarsophalangeal articulation is swollen, hyperemic, hot to touch, painful on movement. A. Esophagoscopy with biopsy In blood: erythrocytes - 5, 1·1012/l, Нb- 155 g/l, B. Urea breath test for H. pylori leukocytes- 13, 0 · 109/l, ESR- 50 mm/hour, C. Diurnal variations of ECG parameters CRP- 46 mg/dl, uric acid - 720 mcmol/l. X-ray D. Esophageal pH monitoring of feet articulations: osteoporosis, narrowing E. Abdominal US of interarticular spaces, numerous punched- 97. After significant physical exertion a 66- out erosions. Make the preliminary diagnosis: year-old man with deep vein thrombosis A. Gout of the extremities developed shortness of B. Osteoarthritis breath, intense pain in the chest on the left, C. Reactive arthritis marked palpitations. The patient’s condition D. Rheumatoid arthritis is grave, his face is cyanotic, the cervical E. Psoriatic arthritis veins are swollen, BP is 60/40 mm Hg. What investigation method would be the most 101. A patient has gradually lost consciousness. advisable in this case? The skin is pale and dry. There is smell of ammonia from the mouth. Respirations are A. Selective angiopneumography deep and noisy. Heart sounds are muffled, B. Chest X-ray pericardial friction rub is present. Blood C. Echocardiography pressure is 180/130 mm Hg. Blood test: Нb- D. Magnetic resonance imaging of the chest · 9 E. Fiber-optic bronchoscopy 80 g/L, leukocytes - 12 10 /L, blood glucose - 6.4 mmol/L, urea - 50 mmol/L, creatinine 98. A 32-year-old woman complains of - 1200 mcmol/L, blood osmolality - 350 general fatigue, low-grade fever persisting mOsmol/kg H2O. No urinary excretion. Make for 4 months, lumbar pain, and dysuria. the diagnosis: Anamnesis includes frequent acute respiratory diseases, overexposure to cold, low-calorie A. Uremic coma diet, a case of pulmonary tuberculosis in B. Hyperglycemic coma childhood. Clinical urine analysis: pH- 4.8, C. Acute renal failure leukocyturia, hematuria. Complete blood D. Acute disturbance of cerebral circulation count: leukocytosis, lymphocytosis, increased E. Hyperosmolar coma ESR. Urography concludes: dilatation of renal 102. A 26-year-old woman has been pelvis and calyceal system of both kidneys, undergoing treatment for community-acquired foci of calcification in the projection of right pneumonia for 10 days. It is known that her kidney parenchyma. What is the most likely Терапевтичний профiль 15 husband had been treated for drug addiction. 106. A 38-year-old woman has been working Sequential intravenous administration of as a milker for 15 years. She made Amoksiklav (Amoxicillin+Clavunate) + an appointment with the doctor due to Levofloxacin combination and vancomycin in development of red rashes on her hands, the prescribed dosage was ineffective. Within predominantly in the interdigital space. The the last two days the patient’s dyspnea and rashes are weeping, itching, and expanding intoxication acutely exacerbated, bilateral on her skin. Examination of her hands shows pulmonary infiltrates are observed. What her nail plates to be yellow and brittle. These is the most likely cause of the medication presentations aggravate during work. Make ineffectiveness? the provisional diagnosis: A. HIV infection and pneumocystic pneumonia A. Occupational eczema B. Tuberculosis mycobacterium infection with B. Scabies development of tuberculosis C. Pemphigus C. Idiopathic fibrosing alveolitis D. Pyoderma D. Infection with polyresistant bacterial strains E. Dermatophytosis E. Cancer metastases in the pulmonary tissues 107. An 18-year-old patient complains of skin 103. A 64-year-old man suddenly sensed rash. The patient has been suffering from pain in his occipital area, dizziness, general this condition for 5 years. The first instance weakness. He has a 15-year-long history of of this disease occurred after a car accident. hypertension. Objectively the skin and mucosa Objectively: the patient presents with papular are of normal color. Auscultation reveals rash covered in silvery scales, ”thimble” vesicular respiration across the lung surface. symptom (small pits on the nails), affected At the cardiac apex the I heart sound is joints. What is the most likely diagnosis? weakened, the II heart sound is accentuated over the aorta. Pulse is 84/min., blood pressure A. Psoriasis is 180/100 mm Hg. Other body organs and B. Panaritium systems are unaffected. What drug should be C. Onychomycosis prescribed in the first place? D. Lupus erythematosus E. Rheumatism A. Captopril B. Ramipril 108. A woman complains of frequent, liquid C. Urapidil stool (up to 9-10 times per day) with mucus D. Perindopril and blood admixtures, dull pain in the E. Amlodipine , weight loss of 4 kg within the last year. Objectively: malnutrition, 104. During winter epidemics of influenza dry skin, low turgor, aphthous stomatitis. caused predominantly by virus А/California/04/2009 The stomach is soft, the sigmoid colon is (H1N1), on the 2nd day after the disease onset spastic and painful on palpation. Occult a 30-year-old hospitalized man presented with blood test is positive. Fibrocolonoscopy: high fever, dry cough, myalgia, headache, and edema, hyperemia, mucosal granulation, general weakness. What should be prescribed pseudopolyps, small ulcers with irregular as etiotropic treatment in this case? edges. Make the diagnosis: A. Neuraminidase inhibitors (Oseltamivir) A. Nonspecific ulcerative colitis B. Antibiotics B. Chronic enterocolitis C. Immunoglobulin C. Colon cancer D. Interferon inducers D. Irritable bowel syndrome E. Acyclovir E. Crohn’s disease (regional enteritis) 105. A 57-year-old patient complains of 109. A 48-year-old woman has been dyspnea at rest. The patient presents with hospitalized due to development of orthopnea, acrocyanosis, bulging cervical tachysystolic atrial fibrillation. She has lost veins. On percussion: dull sound over the lower 5 kg of body weight within 2 months. On lung segments; on auscultation: no respiratory palpation there is a node in the left lobe of murmurs. Heart rate is 92/min. Right-sided the thyroid gland. What pathology resulted in cardiac dilatation is observed. The liver is the development of this condition? enlarged by 7 cm. Shins are swollen. Pleural effusion is suspected. What indicator would A. Toxic nodular goiter confirm the presence of transudate in this case? B. Aterosclerotic cardiosclerosis C. Chronic thyroiditis A. Total protein content in the pleural fluid D. Nontoxic nodular goiter below 25 g/l E. Autoimmune thyroiditis B. Presence of atypical cells C. Total protein content in the pleural fluid 110. A 48-year-old woman developed above30g/l insomnia, depressive mood, anxiety, fears D. Specific gravity exceeding 1015 and suicidal thoughts after the death of her E. Positive Rivalta’s test husband that occurred one month ago. During Терапевтичний профiль 16 her stay in the hospital she speaks in a low A. Neuropathy of the facial nerve voice, is depressed, anxious, avoids sleeping, B. Neuropathy of the trigeminal nerve refuses to eat. What medications should be C. Trigeminal ganglionitis prescribed in this case? D. Neuropathy of the oculomotor nerve E. Ischemic stroke A. Antidepressants B. Antipsychotics 115. A 37-year-old man working as a typesetter C. Group B vitamins in a print shop complains of rapid fatigability, D. Nootropics paroxysmal attacks of stomachache, weak E. Anticonvulsants drooping hands. Examination of neurological status revealed hypotrophy of the forearm 111. A 32-year-old woman complains of muscles. Carporadial reflexes are sharply episodes of intense fear that occur without weakened. Sensitivity is not disturbed. visible cause and last for 10-20 minutes; the Gums present with dark blue border. What episodes are characterized by rapid pulse, neurological pathology is it? sweating, labored breathing, and vertigo. Specify the likely diagnosis: A. Lead polyneuropathy B. Guillain-Barre syndrome (postinfectious A. Panic disorder polyneuritis) B. Paranoid syndrome C. Shingles C. Manic syndrome D. Ulnar neuropathy D. Simple schizophrenia E. Brachial plexitis E. Claustrophobia 116. A 44-year-old woman has undergone 112. A 39-year-old man suffers from chronic subtotal thyroid resection due to diffuse adrenal insufficiency and receives replacement toxic goiter. On the second day after the glucocorticoid therapy (hydrocortisone - 15 surgery the patient’s condition deteriorated; mg/day). He is to undergo elective surgery she developed palpitations, dyspnea, sweating, for calculous cholecystitis. What medication and diarrhea, and became fearful The patient adjustment should be made on the day of the is anxious, her skin is moist and hot to the surgery to prevent the development of acute touch. Her temperature is 39.2oC. Heart adrenal insufficiency? sounds are muffled, tachycardia is observed, pulse is 160/min., blood pressure is 85/40 mm A. Increase the dosage by 2-3 times Hg. The stomach is soft and painless. What B. Cancel the drug for the day of the surgery should be measured to clarify the patient’s C. Add mineralocorticoid condition? D. Add antibiotic E. Prescribe large volume intravenous fluid A. Thyroid hormones infusion B. Blood sugar C. 17-KS and 17-OCS in urine 113. A resuscitation unit received a 46-year-old D. Blood epinephrine and norepineprine woman, who has been suffering from diabetes E. Serum transaminases mellitus type 1 for approximately 30 years. Objectively: the skin is pale, heart sounds 117. The dermatologist has an appointment are weakened, BP is 170/100 mm Hg, lower with a 30-year-old man that complains limbs are markedly swollen. Blood creatinine of severely itching rashes that especially - 1125 mcmol/l, urea - 49,6 mmol/l, potassium disturb him at night. The rashes developed - 6.3 mmol/l, glucose - 7,6 mmol/l, glomerular 2 weeks ago, after he had returned from filtration rate - 5 ml/min. What treatment is a travel. Objectively on the lateral surfaces indicated for the patient in the first place? of his fingers, hands, wrists, elbows, lower abdomen, genitals, and thighs there are paired A. Hemodialysis papulovesicles, single pustules, and scratch B. Kidney transplantation marks. What disease can be suspected? C. Hemofiltration D. Enterosorption A. Scabies E. Conservative detoxification therapy B. Pyoderma C. Dermatitis 114. After a long drive with the window open D. Eczema a man developed facial asymmetry; he cannot E. Shingles close his right eye, his right nasolabial fold is smoothed out, movements of expression are 118. A 28-year-old man, a teacher, after absent on the right, there is a disturbance emotional stress developed painful muscle of gustatory sensation in the tongue on the spasms in his right hand that occur during right. No other neurological pathologies were writing; now he has to hold the pen between detected. What disease can be provisionally the second and third fingers. He has no diagnosed in this patient? problems with typing or writing on the blackboard; no other motor disturbances or neurological pathologies are detected. What is the most likely diagnosis? Терапевтичний профiль 17

122. A 60-year-old man presents with A. Writer’s cramp subcompensated viral liver cirrhosis (HCV), B. Cortical agraphia Child-Pugh class B. What tactics should be C. Parkinsonism chosen regarding the vaccination against D. Neuropathy of the right ulnar nerve influenza in this case? E. Neuropathy of the right radial nerve A. Scheduled yearly vaccination 119. A 43-year-old man, who has been B. In case of influenza outbreak abusing alcohol and suffering from pulmonary C. Combined with antiviral drugs tuberculosis, in the course of two weeks D. Contraindicated due to disease progression gradually developed general weakness, stage, as shown by Child-Pugh class headache, diplopia, vomiting. Objectively: E. Contraindicated due to elderly age of the ptosis on the left, anisocoria S>D, exotropia patient of the left eye, neck stiffness; Kernig’s and Brudzinski’s signs are positive. In 123. A 26-year-old man complains of chills, cerebrospinal fluid: lymphocytic pleocytosis, rhinitis, dry cough, and fever up to 38oC. low glucose, precipitation of cerebrospinal Examination shows him to be in a moderately fluid resulted in production of fibrin film. What severe condition; there are small pale pink is the most likely diagnosis? non-merging spots on the skin of his back, abdomen, and extremities. Palpation reveals A. Tuberculous meningitis enlarged occipital and axillary lymph nodes. B. Subarachnoid hemorrhage No information about vaccination history C. Brainstem encephalitis could be obtained. What is the likely etiology D. Acute myelitis of this disease? E. Basal arachnoiditis A. Rubella virus 120. A 45-year-old woman has been suffering B. Epstein-Barr virus from rheumatoid arthritis for 10 years and C. Streptococcus takes methotrexate twice a week. What D. Mumps virus statement regarding vaccination against E. Neisseria meningitis pneumococci (23-valent vaccine) would conform to the recommendations for the 124. A 40-year-old man developed fever up management of rheumatoid arthritis issued by to 37.5oC and macular rash 10 days after the the European League Against Rheumatism in first dose of MMR (Measles-Mumps-Rubella) 2010? vaccine was administered. The vaccination was considered necessary as there was a measles A. Vaccination is recommended outbreak in the city and the patient had not B. Vaccination is not recommended received MMR vaccination in his childhood. Is C. Vaccination is contraindicated to the patients revaccination with MMR vaccine possible? who take methotrexate D. Vaccination is contraindicated in cases when A. Possible inflammatory process is active B. Forbidden E. Vaccination necessitates increase in the C. After a course of glucocorticoids treatment dosage of the long-term medicines D. Simultaneously with antihistamines E. Under supervision in the infectious diseases 121. A 65-year-old woman was diagnosed with inpatient ward the following: chronic rheumatic heart disease, I degree of rheumatic activity; combined mitral 125. A 26-year-old man is undergoing a heart disease with prevalence of III degree regular check-up. One year ago he had stenosis; heart failure IIA with retained left a case of tonsillar diphtheria complicated ventricular ejection fraction, functional class with myocarditis. Presently his condition is III (NYHA). What tactics of vaccination satisfactory, no signs of cardiovascular failure; against respiratory infections should be ECG shows first-degree atrioventricular block. chosen to provide secondary prevention of What vaccine was administered to this man exacerbations and to avoid heart failure according to his age? decompensation in this patient? A. Adsorbed diphtheria tetanus vaccine A. Scheduled yearly vaccination against (modified) influenza and pneumococci B. Acellular DPT vaccine B. Vaccination should be combined with C. Tetanus anatoxin antibiotic administration D. Oral polio vaccine (OPV) C. Vaccination is contraindicated due to severe E. BCG vaccine heart failure D. Any vaccination is contraindicated due to elderly age of the patient E. Any vaccination is contraindicated due to mitral valve disease Хiрургiчний профiль 18

1. During medical examination a cadet 5. On the 4th day after recovery from a cold in the naval college was detected to have a patient was hospitalized with complaints a painless dense ulcer 1.5x0.5 in size in his of solitary spittings of mucoid sputum. On perianal area at the 2 o’clock position. The the 2nd day there was a single discharge ulcer floor resembles ”old fat”. What is the of about 250 ml of purulent blood-streaked provisional diagnosis? sputum. Objectively: the patient’s condition is moderately severe. Respiratory rate - 28- A. Hard syphilitic chancre of the rectum 30/min., Ps- 96/min., BP- 110/70 mm Hg. B. Rectal fissure Respiration over the left lung is vesicular, C. Rectal fistula over the right lung - weakened. There are D. Anal cancer various moist crackles over the lower lobe E. Anal crypt suppuration and amphoric breath sounds near the angle of scapula. What is the most likely diagnosis? 2. A 10-year-old boy, who was outdoors in windy and cold weather, developed moderate A. Acute pulmonary abscess pain and tingling in his fingers and toes. When B. Exudative pleuritis he had returned home, his parents noticed that C. Acute focal pneumonia the tips of his fingers and toes were white D. Pleural empyema and their sensitivity was lost. The affected E. Pyopneumothorax areas are warming up, the fingers are tingling and in pain. Skin pallor changed into redness, 6. A 65-year-old woman on abdominal tingling stopped, slight itching and swelling of palpation presents with a tumor in the the fingers appeared. Determine the frostbite and above it; the tumor is degree in this child: 13x8 cm in size, moderately painful, non- mobile, pulsing. On auscultation systolic A. Frostbite of the I degree murmur can be observed. What is the most B. Perniosis likely diagnosis? C. Frostbite of the II degree D. Frostbite of the III degree A. Abdominal aneurysm E. Frostbite of the IV degree B. Gastric tumor C. Arteriovenous aneurysm 3. 4 days after a patient received a gunshot D. Tricuspid insufficiency wound to the soft tissues of middle third E. Bicuspid insufficiency of the thigh, his condition suddenly began deteriorating. There are complaints of bursting 7. After a case of purulent otitis a 1-year-old pain in the wound; pain has been increasing boy has developed pain in the upper third of during the last 12 hours. Edema of skin the left thigh, body temperature up to 39oC. and hypodermic tissue quickly grows. Body Objectively: swelling of the thigh in its upper temperature is 38,2oC, heart rate is 102/min. third and smoothed out inguinal fold. The limb The wound edges gape, are dull in color; is in semiflexed position. Active and passive the muscles, viable as of day before, now movements are impossible due to severe pain. protrude into the wound, look boiled, are dull What diagnosis is the most likely? in color, have dirty-gray coating, and fall apart when held with forceps. What infection has A. Acute hematogenous osteomyelitis developed in the wound? B. Acute coxitis C. Intermuscular phlegmon A. Anaerobic D. Osteosarcoma B. Aerobic gram-negative E. Brodie’s abscess C. Putrid D. Aerobic gram-positive 8. A 74-year-old patient was delivered into E. Diphtheria of the wound admission room with clinical presentations of acute deep vein thrombosis of the shin. What 4. A patient received flame burns of both symptom is the most typical of this pathology? hands. On the dorsal and palmar surface of the hands there are blisters filled with serous A. Homans’ sign fluid. The wrist joint region is hyperemic. B. Rovsing’s sign The forearms were not injured. What is the C. Courvoisier’s sign provisional diagnosis? D. Mayo-Robson’s sign E. Grey Turner’s sign A. II degree flame burn of the hands, 4% of body surface area 9. A 50-year-old patient was delivered to a B. II degree flame burn of the hands, 2% of hospital with complaints of blood traces in body surface area urine. Urination is painless and undisturbed. C. IIIa degree flame burn of the hands, 4% of Macrohematuria had been observed for 3 body surface area days. Objectively: kidneys cannot be palpated, D. III degree flame burn of the hands, 4% of suprapubic area is without alterations, body surface area external genitalia are non-pathologic. On E. IIb degree flame burn of the hands, 2% of rectal investigation: prostate is not enlarged, body surface area painless, has normal structure. Cystoscopy revealed no alterations. What is the most likely Хiрургiчний профiль 19 diagnosis? 14. A 58-year-old patient complains of pain in the lower left extremity, which aggravates A. Renal carcinoma during walking, and sensation of cold and B. Bladder tuberculosis numbness in the both feet. The patient has C. Varicocele been suffering from this condition for 6 years. D. Dystopic kidney Objectively: the skin is pale and dry, with E. Necrotic papillitis hyperkeratosis. On the left shin hair is scarce. Pulse cannot be detected over the pedal and 10. A man complains of constant dull pain in popliteal arteries and is weakened over the the and suprapubic area, weak flow femoral artery. On the right limb pulsation of of urine, frequent difficult painful urination, the popliteal artery is retained. What is the nocturia. The patient has been suffering from most likely diagnosis? this condition for several months, during which urination was becoming increasingly difficult, A. Atherosclerosis obliterans of the lower and pain in the perineum has developed. On extremities rectal examination: the prostate is enlarged B. Obliterating endarteritis (mainly its right lobe), dense, asymmetrical, C. Femoral artery thrombosis central fissure is smoothed out, the right lobe D. Raynaud’s disease is of stony density, painless, tuberous. What E. Buerger’s disease (thromboangiitis disease is it? obliterans) A. Prostate cancer 15. A 47-year-old woman came to the B. Prostate sclerosis admission room with complaints of general C. Urolithiasis, prostatolith of the right lobe weakness, dizziness, vomiting with blood D. Prostate tuberculosis clots. Condition onset was 3 hours ago. The E. Chronic congestion prostatitis patient has no preceding illnesses. Blood pressure is 90/60 mm Hg, pulse is 106/min., 11. A 17-year-old young man complains of of poor volume. The abdomen is soft, with general weakness, trismus, twitching of the mild tenderness in the epigastrium. Blood muscles in his left shin. 7 days ago he pierced · 12 his foot with a nail. Objectively: at the sole test: erythrocytes - 2.1 10 /L, Нb- 70 g/L, of the foot there is a wound, 0,3х0,2 mm in hematocrit - 28%. What tactics should the size, with small amount of serous-purulent doctor on duty choose? discharge, the skin around the wound is A. Consult the surgeon hyperemic. What is the most likely diagnosis? B. Refer the patient to the family doctor A. Tetanus C. Give spasmolytics B. Phlegmon D. Perform gastric lavage C. Osteomyelitis E. Make an appointment for colonoscopy D. Infected wound 16. A 52-year-old patient complains of pain E. Erysipelas in the right part of her chest, dyspnea, 12. A patient with trauma of the lower third of cough with large amounts of foamy sputum the forearm volar surface caused by a glass emitting foul smell and resembling ”meat shard came to a first-aid center. Objectively: slops”. Objectively: the patient’s condition is flexion of the IV and V fingers is impaired, grave, cyanosis is observed, breathing rate is sensitivity of the inner dorsal and palmar 31/min., percussion sound above the right lung surfaces of the hand and IV finger is decreased. is shortened, auscultation revealed various What nerve is damaged? moist rales (crackles). What is the most likely diagnosis? A. Ulnar B. Radial A. Lung gangrene C. Median B. Lung abscess D. Musculocutaneous C. Empyema of pleura E. Axillary D. Multiple bronchiectasis E. Chronic pneumonia 13. A man diagnosed with closed-angle 17. A man complains of sore throat on the left, glaucoma, grade IIa, of the right eye is o o registered for regular medical check-ups. In pain in his left ear, t up to 39 C, and nasal the evening an acute glaucoma attack occurred sound of his voice. Disease onset was 5 days in his right eye; an ambulance was called. What ago. Marked trismus and increased salivation emergency aid would be optimal in this case? are observed. The head tilts to the left shoulder. Left side of the soft palate presents A. Pilocarpine, Diacarb (Acetazolamide), lytic with swelling, hyperemia, and infiltration. mixture Retromandibular lymph nodes on the left are B. Atropine eye drops acutely painful on palpation. Otoscopy results C. Antibiotic eye drops, broad-spectrum are normal. Make the diagnosis: D. Sulfacetamide sodium eye drops E. Dexamethasone eye drops Хiрургiчний профiль 20

A. Left-sided peritonsillar abscess A. Laparotomy B. Retropharyngeal abscess B. Laparoscopy C. Parapharyngeal phlegmon C. Cold to the abdomen D. Peritonsillitis on the left D. Abdominal X-ray E. Cervical phlegmon on the left E. Laparocentesis 18. A patient has the second and third degree 22. A 48-year-old woman was arrived to the burns of the 15% of the body surface. On the surgical unit with wounds in her thigh. On 20th day after the trauma the patient presents examination the wound surface has dirty- with sharp increase of body temperature, gray coating with unpleasant sweet smell. general weakness, rapid vesicular respiration; Wound content resembles raspberry jelly. Skin facial features are sharpened, BP is 90/50 mm tissues around the wound are glossy and turgid. Hg, heart rate is 112/min. What complication Palpation reveals moderate crepitation in the is it? tissues. What microflora is the most likely to cause such inflammation? A. Sepsis B. Pneumonia A. Anaerobic clostridial C. Acute intoxication B. Anaerobic non-clostridial D. Purulent bronchitis C. Streptococci E. Anaerobic infection D. Staphylococci E. Blue pus bacillus 19. A patient in the state of clinical death is being resuscitated through mouth-to-mouth 23. A 30-year-old patient was hospitalized artificial pulmonary ventilation and external in an intensive care unit with a diagnosis of cardiac massage. A doctor noticed that air does multiple bee stings. Skin is pale and covered not flow into the patient’s airways and his head with cold sweat. Pulse can be palpated only at and torso are positioned at the same level. Why the carotid arteries and is 110/min.; breathing is artificial respiration ineffective in the given rate is 24/min., rhytmical, weakened. What case? drug must be administered immediately? A. Tongue retraction A. Epinephrine hydrochloride B. Low breathing volume B. Prednisolone C. External cardiac massage C. Norepinephrine hydrochloride D. Probe is absent from the stomach D. Dopamine E. The patient’s mouth is too small E. Tavegyl (Clemastine) 20. A patient complains of suppuration from 24. A 46-year-old woman has been hospitalized the ear and impaired hearing of the left ear, with open fracture of the left thigh in its middle which have been observed for the past 6 third. She underwent the surgery - fixation with years. The patient had periodical headaches, extraosseous osteosynthesis plates. On the 4th general indisposition, fever. Objectively: day after the surgery she developed pain in otoscopy of the external auditory meatus the wound, body temperature rose over 39oC. revealed mucopurulent odorless substance. What measures should be taken in this case? The eardrum is of normal color, with central perforation. What is the most likely diagnosis? A. Undo the sutures, drain the wound, and prescribe antibiotics A. Chronic mesotympanitis B. Prescribe broad spectrum antibiotics and B. Otosclerosis hormonal agents C. Acute otitis media C. Administer antibiotics intraosseously and D. Chronic epitympanitis hypothermia locally E. Chronic sensorineural hearing loss D. Inject antibiotics into the area surrounding the wound, prescribe spasmolytics and 21. In 2 hours after a traffic accident a 28-year- analgesics old man in grave condition was delivered to a E. Remove the fixation, prescribe hospital. The patient complains of abdominal sulfanilamides pain. He received a blow to the abdomen with the steering wheel. Objective examination 25. A 42-year-old man was delivered to a revealed the following: the abdomen does not surgical in-patient department with complaints participate in respiration, is tense and acutely of icteric skin, pain in the right subcostal area. painful on palpation; abdominal guarding is Biochemical blood analysis: total bilirubin - present, peritoneal irritation signs are positive, 140 mcmol/l, direct bilirubin - 112 mcmol/l. On hepatic dullness is absent. BP is 90/60 mm Hg, US: choledoch duct - 1,4 cm, a concrement is heart rate is 120/min. What further treatment detected in the distal area. Gallbladder is 40 tactics should be chosen? cm, no concrements. What treatment tactics should be chosen? Хiрургiчний профiль 21

A. Endoscopic papillosphincterotomy A. Surgical intervention B. Laparoscopic cholecystectomy B. Administration of spasmolytics C. Laparotomy with choledoch duct drain C. Administration of ACE inhibitors D. Laparotomy with cholecystectomy D. Kidney catheterization E. Threatment in an infectious diseases hospital E. Administration of β-blockers 26. 4 weeks after myocardial infarction a 56- 30. A 49-year-old patient consulted a year-old patient developed acute heart pain, doctor about difficult swallowing, voice marked dyspnea. Objectively: the patient’s hoarseness, weight loss. These symptoms condition is extremely grave, there is marked have been gradually progressing for the cyanosis of face, swelling and throbbing of last 3 months. Objectively: the patient is neck veins, peripheral pulse is absent, the exhausted, there are enlarged supraclavicular carotid artery pulse is rhythmic, 130/min., BP is lymph nodes. Esophagoscopy revealed no 60/20 mm Hg. Auscultation of the heart reveals esophageal pathology. Which of the following extremely muffled sounds, percussion reveals investigations is the most appropriate in this heart border extension in both directions. case? What is the optimal treatment tactics for this patient? A. Computed tomography of chest and mediastinum A. Pericardiocentesis and immediate B. X-ray of lungs thoracotomy C. Multiplanar imaging of esophagus B. Oxygen inhalation D. Radioisotope investigation of chest C. Puncture of the pleural cavity on the left E. Ultrasound investigation of mediastinum D. Conservative treatment, infusion of adrenomimetics 31. A 38-year-old patient has been delivered E. Pleural cavity drainage by an ambulance to a surgical department with complaints of general weakness, indisposition, 27. A 43-year-old woman complains of black stool. On examination the patient is pale, persistent stomachache with recurrent pain there are dotted hemorrhages on the skin of his attacks, nausea, repeated vomiting with torso and extremities. On digital investigation stagnant bowel content, abdominal distension, there are black feces on the glove. Blood test: and flatulence. She has been presenting with Hb- 108 g/L, thrombocytopenia. Anamnesis these signs for 7 hours. Pulse is 116/min. states that similar condition was observed 1 The tongue is dry and brown. The abdomen year ago. Make the diagnosis: is symmetrically distemded, soft, painful. Percussion reveals tympanitis. On auscultation A. Thrombocytopenic purpura there are bowel sounds with metallic overtone, B. Hemophilia splashing, and dripping. Make the diagnosis: C. Bleeding from an ulcer D. Rectal tumor A. Acute intestinal obstruction E. Nonspecific ulcerative colitis B. Acute necrotizing pancreatitis C. Acute destructive cholecystitis 32. A 3-year-old child presents with sharp D. Acute erosive gastritis deterioration of his general condition. He has E. Acute nonspecific colitis a history of purulent otitis. His temperature is now 38.5oC. The left leg is pressed to the 28. An infant cries during urination, the torso, active movements are absent, the lower foreskin swells and urine is excreted in third of the thigh and knee joint are thickened, drops. What approach to treatment should be hyperemic, with localized fever. Axial load chosen? leads to acute discomfort of the patient. What is the most likely diagnosis? A. Create an opening into the foreskin cavity B. Prescription of α-adrenergic blocking agents A. Epiphyseal osteomyelitis on the left C. Prescription of antispasmodic agents B. Left hip fracture D. Urinary bladder catheterization C. Rheumatoid arthritis E. Epicystostomy D. Osteogenic sarcoma E. Hygroma of the knee 29. A 20-year-old patient complains of pain in the left lumbar region, arterial pressure rise 33. A patient after a blow to the head up to 160/110 mm Hg. US revealed that the developed general symptoms of cerebral structure and size of the right kidney were disturbance, nausea, vomiting, focal signs - within age norms, there were signs of the 3rd hemi-hyperreflexia S>D, hemihyperesthesia degree hydronephrotic transformation of the on the left, marked meningeal syndrome. left kidney. Doppler examination revealed Neither cranial X-ray nor computer an additional artery running to the lower tomography revealed any pathologies. What pole of the kidney. Excretory urogram shows examination method would allow making and a narrowing in the region of ureteropelvic clarification of the diagnosis? junction. Specify the treatment tactics: Хiрургiчний профiль 22

A. Lumbar puncture Cytoscopy detected hemiatrophy of the B. Echoencephalography urinary bladder trigone, the right ureteral C. Electroencephalography orifice is not detected. What pathology is it? D. Angiography E. Pneumoencephalography A. Agenesis of the right kidney B. Dystopia of the right kidney 34. A burn victim with flame burns of the C. Hypoplasia of the right kidney IIIA-B and IV degrees on his face, neck, and D. Agenesis of the right ureter anterior surface of the thorax was brought E. Ectopic right ureteral orifice into the admission room. The hairs in his nostrils are burnt, his labial and glossal 38. A 64-year-old patient has been hospitalized mucosa are gray-white. The voice is hoarse; with complaints of progressive jaundice respirations are frequent and shallow; the that developed over 3 weeks without pain patient has trumpet-like cough that produces syndrome and is accompanied by general soot-streaked sputum. The signs of respiratory weakness and loss of appetite. Objectively: failure were progressing, while the patient was temperature is 36,8oC, heart rate is 78/min, being transported into the intensive care unit. abdomen is soft and painless, peritoneum What emergency care must be provided to this irritation symptoms are not detected, patient? palpation reveals sharply enlarged tense gallbladder. What disease can be characterised A. Intubation of the trachea and mechanical by these symptoms? ventilation B. Tracheostomy A. Cancer of pancreas head C. Administration of bronchial spasmolytics B. Duodenal ulcer D. Administration of respiratory analeptics C. Acute cholecystitis E. Inhalation of moisturized oxygen D. Chronic cholecystitis E. Cholecystitis caused by lambliasis 35. The body of a 24-year-old woman with probable signs of poisoning has been found 39. A 37-year-old patient complains of pain on the street. Forensic medical examination in the spinal column, reduced mobility. The was requested by an investigator during condition persists for 7 years. ”Sway back” is examination of the site and the body. observed, there is no movement in all spinal According to the Criminal Procedure Code regions. On X-ray: ”bamboo spine” is detected. currently in force in Ukraine, forensic medical What is the most likely diagnosis? examination is required when it is necessary to determine the: A. Ankylosing spondylitis B. Osteochondrosis A. Cause of death C. Spondylitis deformans B. Manner of death D. Tuberculous spondylitis C. Time of death E. Spondylolisthesis D. Mode of death E. Mechanism of death 40. A 54-year-old patient complains of weakness, jaundice, itching skin. Disease onset 36. A 27-year-old man was hospitalized in was 1.5 months ago: fever up to 39oC appeared severe condition 50 minutes after receiving a at first, with progressive jaundice developed 2 penetrating wound to the left side of the chest. weeks later. On hospitalisation jaundice was Objectively the patient is in a stupor, his skin severely progressed. Liver cannot be palpated. is pale and acrocyanotic. Pulse is 120/min., of Gallbladder is enlarged and painless. Blood poor volume, weak. Blood pressure is 80/40 bilirubin is 190 mcmol/L (accounting mainly mm Hg. Heart sounds are muffled, cardiac for direct bilirubin). Stool is acholic. What borders are markedly expanded. In the III is the most likely reason for jaundice in this intercostal area along the parasternal line on patient? the left there is a stab-incised wound. Plain chest X-ray shows enlarged heart shadow A. Mechanical jaundice with smoothed out waist of the heart, there is B. Hepatocellular jaundice hemothorax on the left to the 5th rib. What C. Hemolytic jaundice contributes the most to the severity of the D. Caroli syndrome patient’s condition? E. Gilbert’s syndrome A. Cardiac tamponade 41. A 23-year-old man came to the surgeon B. Acute heart failure with complaints of pain, redness of the skin, C. Cardiac rhythm disturbance and swelling in the area of his proximal D. Blood loss interphalangeal joint of the III finger on the E. Hemothorax and acute respiratory failure right hand. Six days ago he pricked his finger with a wire. Objectively the III finger on the 37. During regular examination of a 2-year- right hand is swollen, hyperemic, prominent in old boy, he presents with enlarged left kidney, the projection of interphalangeal joint, sharply painless on palpation. The right kidney painful on touch and during movements. was undetectable on palpation. Excretory Finger mobility is reduced. Fluctuation sign urography shows no contrast on the right. Хiрургiчний профiль 23 is present. What diagnosis corresponds to the A. Esophageal tumor given clinical presentation? B. Achalasia cardiae C. Esophageal burns A. Articular panaritium D. Diverticulum B. Bone panaritium E. Barrett esophagus C. Subcutaneous panaritium D. Pandactylitis 46. A 45-year-old woman underwent one E. Finger furuncle year ago mastectomy followed by chemo- and radiation therapy. She now complains of 42. A 30-year-old patient was in a car accident. dyspnea at rest and temperature up to 37.2oC. He is unconscious, pale, has thready pulse. In Her general condition is severe, acrocyanosis the middle third of the right thigh there is is observed. The right side of her chest an extensive laceration with ongoing profuse practically does not participate in respiration. external arterial bleeding. What urgent actions Percussion reveals a dull sound below the must be taken to save the life of the patient? 3rd rib; auscultation detects acute weakening of the respiratory sounds. Pleural puncture A. Tourniquet above the wound of the right on the right has yielded a large amount of thigh hemorrhagic exudate. What complication has B. Tourniquet below the wound of the right developed in the patient? thigh C. Artificial lung ventilation A. Carcinomatous pleuritis D. Precordial thump B. Acute pleural empyema E. Application of plaster bar C. Acute right-sided pleuropneumonia D. Right lung abscess 43. A 25-year-old patient has been E. Pulmonary embolism admitted to the hospital with the following problems: weakness, sweating, itching, weight 47. A patient with signs of general loss, enlarged submandibular, cervical, overexposure to cold presenting with local axillary, inguinal lymph nodes. Objectively: frostbites of fingers has been delivered into an hepatomegaly. Lymph node biopsy revealed admission room. Objectively: conscious, inert, giant Berezovsky-Reed-Sternberg cells, speech is slow, the skin of the face is cold, body polymorphocellular granuloma composed temperature is 34oC, heart rate is 68/min. What of lymphocytes, reticular cells, neutrophils, would be the actions of a doctor on call? eosinophils, fibrous tissue, and plasma cells. What is the most likely diagnosis? A. Hospitalize the patient to the surgical department A. Lymphogranulomatosis (Hodgkin’s B. Hospitalize the patient to the therapeutics lymphoma) department B. Lymph node tuberculosis C. Hospitalize the patient to the traumatology C. Lymphoreticulosarcoma department D. Cancer metastases to the lymph nodes D. Let the patient go home E. Macofollicular reticulosis E. Refer to a family doctor on the next day 44. A 68-year-old man complains of inability 48. A 24-year-old patient had been delivered to urinate for a day. On attempt of urinary to a thoracic department with a chest injury, bladder catheterization there was detected a fracture of the IV, V, VI ribs on the right. rough stricture in the membranous portion of Plan radiography showed the fluid level in the the urethra. What first aid tactics should be pleural cavity to be reaching the III rib on the applied in this case? right. Puncture contained blood clots. What is the optimal treatment tactics? A. Troacar cystostomy B. Adenomectomy A. Emergency thoracotomy C. Optical internal urethrotomy B. Pleural puncture D. α-adrenoblockers C. Thoracentesis and thoracostomy E. Urinary bladder catheterization D. Hemostatic therapy E. Medical thoracoscopy 45. Heart X-ray of a 31-year-old man has revealed the following: with tightly filled 49. A 74-year-old patient visited a urologist opacified esophagus there is a marginal filling with complaints of pain above the pubis and defect in its middle third on the posterior wall; inability to urinate for 8 hours. At home he the defect is 1,8x1,3 cm in size with clear oval had taken antispasmodics and had a warm bath border. Mucosal folds are retained and envelop but no improvement occurred. Objectively: the defect; wall peristalsis and elasticity are not abdomen is soft and painful above the pubis; affected. There are no complaints regarding dullness of percussion sound is observed above the condition of the patient’s alimentary canal. the pubis. Murphy’s (Pasternatski’s) punch Make the provisional diagnosis: sign is negative on the both sides. What condition does the patient have? Хiрургiчний профiль 24

A. Acute urinary retention A. Sepsis B. Paradoxal ischuria B. Purulent-resorptive fever C. Chronic urinary retention C. Trombophlebitis D. Anuria D. Meningitis E. Oliguria E. Pneumonia 50. During preventive examination a 58-year- 54. A 28-year-old woman complains of girdle old man on chest X-ray presents with multiple pain in her epigastric and left subcostal areas globular pale shadows 3 cm in diameter within with irradiation to the back, nausea, and parenchyma of the both lungs. Examination vomiting without relief. On examination a in the oncologic hospital: the primary focus is surgeon observes stomach distension and not found; transbronchial biopsy with cytologic meteorism. There are positive Mondor’s, investigation detected cells of glandular Mayo-Robson’s, and Cullen’s symptoms. What neoplasm. What tactics should the physician is the most likely diagnosis? choose? A. Acute pancreatitis A. Polychemotherapy courses B. Acute cholecystitis B. Exploratory laparotomy C. Acute intestinal obstruction C. Exploratory thoracotomy D. Aortic dissecting aneurysm D. Laparoscopy E. Splenic infarction E. Symptomatic treatment at home 55. A man complains of high fever, pain in the 51. A 35-year-old man complains of persisting area of his right mastoid bone, and purulent enlargement of his peripheral lymph nodes discharge from the right ear. One week ago that cause him no discomfort. The case he had a case of URTI. Objectively the right history states that the first lymph nodes auricle protrudes, the skin behind the ear is to enlarge were cervical, supraclavicular, hyperemic and pastose; on palpation of the and axillary; new groups of lymph nodes mastoid bone the pain intensifies; the auditory emerged. Objectively the lymph nodes are soft meatus is filled with thick pus, posterosuperior and elastic on palpation, enlarged, painless, meatal wall sags; the tympanic membrane is not fixed to the surrounding tissue. What red and perforated. Make the diagnosis: examination method would be the most informative for early diagnostics of this A. Acute mastoiditis disease? B. Furuncle of the external auditory meatus C. Acute otitis media A. Needle biopsy D. Acute otitis externa diffusa B. Magnetic resonance tomography E. Exacerbation of chronic mesotympanitis C. Radioisotope scanning of the skeleton D. Ultrasound 56. A 28-year-old man after car accident E. X-ray received a wound to the right side of his chest inflicted by a sharp metal object. A foamy 52. A woman in her early- to mid-thirties has liquid flows out from the wound, there are lost her consciousness 3-5 minutes ago. On tympanitis and acutely weakened respirations examination: the skin is pale, no pulse over the in the right. Blood pressure is 70/30 mm Hg, carotid arteries, no spontaneous respiration, pulse is 120/min., Hb is 28 g/L. X-ray shows pupils are dilated; the patient is nonresponsive, collapsed right lung, horizontal fluid level is at presents with atony. The patient’s condition the 3rd rib. What treatment tactics should be can be determined as: chosen? A. Apparent death A. Urgent thoracotomy B. Natural death B. Delayed thoracotomy C. Syncope C. Drain the right pleural cavity D. Brain death D. Apply occlusive dressing to the wound E. Coma E. Conservative therapy 53. A 38-year-old man underwent surgical 57. When her car collided with a tree, a 37- treatment of a wound with a suppuration year-old woman felt sharp pain in her left hip focus. On the 8th day after the procedure the joint. She was unable to get out of the car. wound cleared of purulo-necrotic discharge Her position is forced, the hip is pressed to the and granulations appeared. However, against abdomen, fixed, and rotated inwards; the limb the background of antibacterial therapy, the is flexed in the knee, any attempt to change patient’s body temperature persists as high the position results in sharp pain. Make the as 38.5-39.5oC; chills, excessive sweating, and diagnosis: euphoria are observed in the patient; heart rate is 120/min. What complication of the A. Closed dislocation of the left hip local suppurative inflammatory process can be B. Contusion of the left hip joint suspected? C. Hemarthrosis of the left hip joint D. Arthritis of the left hip joint E. Closed cervical fracture of the left hip Хiрургiчний профiль 25

58. A 22-year-old man at 18:00 developed of treatment should be prescribed in this case? persisting dull pain in the epigastrium. Three hours later nausea appeared, he vomited A. Surgical once. By the morning the pain shifted to the B. Conservative right iliac area. Body temperature rose to C. Physical therapy 38.6oC, developed tachycardia of 110/min. D. Case monitoring On examination there are muscle rigidity and E. Abstain from surgery in favor of complex Bloomberg’s sign (rebound tenderness) in the conservative therapy right iliac area of the anterior abdominal wall. Plain x-ray of the abdomen shows no fluid 62. A 6-year-old girl arrived to the in-patient levels, free air under the diapragm on the right. unit with complaints of enlargement of the Make the diagnosis: lower third of her right thigh. According to the case history, she has been stepping carefully A. Perforation of a gastric ulcer on her right leg and limping for 6 months. B. Renal colic Blood test detected anemia. X-ray of the right C. Acute appendicitis thigh shows a round bone defect with clear D. Acute pancreatitis margins resembling melting sugar in the distal E. Acute cholecystitis metaphysis. What provisional diagnosis can be made in this case? 59. On ultrasound of the thyroid gland, a 47- year-old woman presents with a hypoechoic A. Osteogenic sarcoma of the right femur node 1.6 cm in diameter with blurred margins B. Rheumatoid arthritis of the right knee joint and intranodular hypervascularization. The C. Acute hematogenous osteomyelitis of the doctor suspects thyroid carcinoma. What distal femoral metaphysis on the right method should be used to verify the diagnosis? D. Tuberculous osteitis of the distal femoral metaphysis on the right A. Fine-needle aspiration biopsy E. Giant cell tumor of the right femur B. Thyroid scintigraphy C. Case monitoring 63. A 46-year-old man came to the surgeon’s D. Determine TSH level in the blood office. He complains of twitching sensation E. Positron emission tomography (PET) in the wound on his left foot, insomnia, and anxiety. According to the patient, he 60. A 19-year-old young man complains of received this wound 5 days ago, when he cough with expectoration of purulent sputum accidentally stepped on a glass shard, while on in amount of 100 ml per day, hemoptysis, the beach. He requested no medical assistance. dyspnea, increased body temperature up to Objectively the patient’s general condition is 37.8oC, general weakness, weight loss. The satisfactory, pulse is 75/min., blood pressure is patient’s condition has been persisting for 4 130/80 mm Hg, temperature is 36.9oC. On the years. Exacerbations occur 2-3 times per year. plantar surface of his foot there is a wound 1.5 The patient presents with malnutrition, pale cm long and up to 3 cm deep. The wound edges skin, cyanosis of the , drumstick (clubbed) are moderately hyperemic, no discharge from fingers. Tympanic percussion sound in the the wound is observed. What disease can be lungs, weakened respiration, various numerous suspected in this patient? moist crackles in the lower pulmonary segments on the left can be observed. In A. Tetanus · 12 B. Diphtheria blood: erythrocytes - 3.2 10 /L, leukocytes C. Anthrax · 9 - 8.4 10 /L, ESR- 56 mm/hour. On X-ray: lung D. Fasciitis fields are emphysematous, the left pulmonary E. Phlegmon root is deformed and dilated. What is the most likely diagnosis? 64. A 20-year-old student was brought to the first-aid center. He has a closed fracture of the A. Multiple bronchiectasis of the left lung left forearm and a contused lacerated wound B. Chronic left-sided pneumonia on his left shin. After the patient received C. Chronic abscess of the left lung initial wound management, he presented the D. Left-sided pulmonary cystic dysplasia documents confirming that he has received E. Suppuration of the cyst in the left lung all the necessary preventive vaccination as scheduled. What should the doctor do to 61. An 8-year-old child presents with blood prevent tetanus in this patient? pressure up to 180/100 mm Hg in the upper limbs accompanied by headaches, A. Dynamic case monitoring tinnitus, occasional nosebleeds, and high B. Administration of tetanus immunoglobulin fatigability. On examination there is no C. Administration of anti-tetanus serum pulse over the leg arteries. ECG shows left D. Antibiotic therapy ventricular hypertrophy. MRI-scan shows E. Administration of tetanus toxoid aortic narrowing to 5 mm in the typical place. Coarctation of aorta is diagnosed. What kind Педiатричний профiль 26

1. An 8-year-old boy developed a A. Surgical intervention temperature of 37, 5oC two days after B. Continuation of conservative therapy his recovery from the case of URTI. C. Physiotherapy He complains of suffocation, heart pain. D. Sanatorium-and-spa treatment Objectively: the skin is pale, tachycardia, the E. Physical training I heart sound is weakened, short systolyc murmur in the 4th intercostal area near 5. An infant has been born at the 41st week the left edge of the breastbone. What of gestation. The pregnancy was complicated heart disorder such clincal presentation is with severe gestosis of the second semester. characteristic of? The weight of the baby is 2400 g, the height is 50 cm. Objectively: the skin is flabby, the layer A. Nonrheumatic myocarditis of subcutaneous fat is thin, hypomyotonia, B. Primary rheumatic carditis neonatal reflexes are weak. The internal C. Myocardiodystrophy organs are without pathologic changes. This D. Fallot’s tetrad newborn can be assessed as a: E. Cardiomyopathy A. Full-term infant with intrauterine growth 2. A 7-year-old boy has severe pulmonary retardation mucoviscidosis (cystic fibrosis). He complains B. Premature infant of dyspnea and blood expectoration. C. Immature infant Objectively he presents with lagging physical D. Postmature infant development, acrocyanosis, hepatomegaly, E. Full-term infant with normal body weight drumstick fingers, and nail plates resembling a”clockface”. Provisional diagnosis of chronic 6. During examination a 4-month-old pulmonary heart disease is made. What child with meningococcemia presents with examination would be the most informative acrocyanosis, cold extremities, tachypnea, and for diagnosis confirmation? thready pulse, blood pressure of 30/0 mm Hg, anuria, and sopor. What clinical syndrome is A. Doppler echocardiography it? B. Electrocardiography Septic shock C. Chest X-ray A. Rheography of the pulmonary artery B. Neurotoxicosis D. Exicosis E. Ultrasound of the liver C. D. Encephalic syndrome 3. A 12-year-old boy with hypertrophic E. Acute renal failure cardiomyopathy complains of dyspnea caused by the slightest physical exertion. 7. The right of a newborn is stretched Echocardiography detected asymmetrical along the torso with all its joints extended; the left ventricular hypertrophy, signs of shoulder is rotated inwards, while the forearm pulmonary hypertension, and left ventricular is pronated, the hand is in the position of dilatation, its ejection fraction is 59%. These palmar flexion. Spontaneous movements are developments are indicative of: absent in the shoulder and elbow joints, passive movements are painless. What is the A. Heart failure with preserved ejection most likely diagnosis? fraction Duchenne-Erb palsy, superior proximal B. Heart failure with reduced ejection fraction A. Primary pulmonary hypertension type C. Dejerine-Klumpke palsy, inferior distal type D. Essential hypertension B. Total obstetric palsy E. Symptomatic arterial hypertension C. D. Osteomyelitis of the right humerus 4. A 9-year-old boy has been suffering from E. Poliomyelitis multiple bronchiectasis since he was 3 years 8. A 1-year-old child with a case of URTI old. Exacerbations occur frequently (3-4 suddenly developed noisy respirations with times a year), after conservative therapy difficult inspiration, intercostal retractions, there are short remission periods. The and barking cough on the 2nd night after disease progresses, the child is physically the disease onset. What is the most likely underdeveloped, presents with pale skin, diagnosis? acrocyanosis, deformed nail plates in the shape of ”clock-face”. Bronchography reveals A. Stenosing laryngotracheobronchitis saccular bronchiectases in the lower lobe of B. Acute pulmonary inflammation the right lung. What further treatment tactics C. Bronchial asthma should be chosen? D. Acute bronchitis E. Acute bronchiolitis 9. A 10-year-old boy with symptoms of arthritis and myocarditis was delivered into a hospital. Based on clinical examination the preliminary diagnosis of juvenile rheumatoid Педiатричний профiль 27 arthritis was made. What symptom is the A. Mucoviscidosis (Cystic fibrosis) most contributive for the diagnostics of this B. Acute obstructive bronchitis disease? C. Recurrent obstructive bronchitis D. Pertussis A. Reduced mobility of the joints in the E. Acute obstructive pneumonia morning B. Regional hyperemia of the joints 13. A 10-year-old girl complains of C. Affection of the large joints stomachache that appears and intensifies after D. Enlarged heart she eats rough or spicy food, sour eructation, E. Increased heart rate heartburn, frequent constipations, headaches, irritability. She has been presenting with these 10. A 9-month-old child presents with fever, signs for 12 months. Her meals are irregular cough, dyspnea. The symptoms appeared and consist of dry food. Objectively her diet is 5 days ago after a contact with a person sufficient in calories. The tongue is moist with with URTI. Objectively: the child is in white coating near the root. The abdomen grave condition. Temperature is 38oC, is soft and painful in the epigastrium. What cyanosis of nasolabial triangle is present. method would be optimal for diagnosis- RR- 54/min, nasal flaring during breathing making in this case? is observed. There was percussion dullness on the right below the scapula angle and A. Esophagogastroduodenoscopy tympanic sound over the other areas of lungs. B. Intragastric pH-metry Auscultation revealed bilateral fine moist C. Fractional gastric analysis (Fractional test crackles predominating on the right. What is meals) the most likely diagnosis? D. Phase-contrast X-ray imaging E. Biochemical blood test A. Pneumonia B. URTI 14. A boy was born at 32 weeks of gestation. 2 C. Acute laryngotracheitis hours after the birth he developed respiratory D. Acute bronchitis distress (RD). The RD severity assessed E. Acute bronchiolitis by Silverman score was 5. The respiratory disorders progressed, respiratory failure could 11. A 7-year-old boy has been an inpatient not be eliminated by Martin-Bouyer CPAP for 1.5 months. He had been delivered to (continuous positive airway pressure). X-ray the hospital with complaints of edemas of lungs shows reticular and nodular pattern, all over his body, low urine output, and air bronchogram. What is the most likely headache. Clinical urinalysis: proteins - cause of respiratory distress syndrome? 7.1g/L,leukocytes-1-2inthevision field, erythrocytes - 3-4 in the vision field. A. Hyaline membrane disease During the course of treatment the edemas B. Segmental atelectasis gradually dissipated, headache abated, C. Bronchopulmonary dysplasia diuresis normalized. Daily urine proteins - D. Congenital pulmonary emphysema 3 g/L. Biochemical blood test: total protein - E. Edematous hemorrhagic syndrome 43.2 g/L, urea - 5.2 mmol/L, cholesterol - 9.2 mmol/L. What glomerulonephritis syndrome 15. An 8-year-old child with a 3-year-long isthemostlikelytobepresentinthepatient? history of diabetes was hospitalized in hyperglycemic coma. Specify the initial dose A. Nephrotic of insulin to be administered: B. Nephritic C. Isolated urinary A. 0,1-0,2 U/kg of body weight per hour D. Hematuric B. 0,05 U/kg of body weight per hour E. Mixed C. 0,2-0,3 U/kg of body weight per hour D. 0,3-0,4 U/kg of body weight per hour 12. The mother of a 3-month-old child came E. 0,4-0,5 U/kg of body weight per hour to a family doctor with complaints of her child being physically underdeveloped and 16. A 5-year-old child that contacts with suffering from cough attacks and dyspnea. viral hepatitis in the kindergarten presents Anamnesis: the child is the result of the with increased body temperature up to second full-term pregnancy with the risk of 38oC, weakness, low appetite, single case of miscarriage (the first child died of pulmonary vomiting, dull pain in the subcostal area on pathology at the age of 4 months, according the right. The child is provisionally diagnosed to the mother). Body mass at birth is 2500 g. with viral hepatitis. What examination Cough attacks were observed from the first would be the most informative for diagnosis days of life, twice the child was treated for confirmation? bronchitis. Considering the severity of the child’s condition the doctor made the referral for hospitalization. What diagnosis was most likely stated in the referral? Педiатричний профiль 28

A. ALT activity in blood A. Ciprofloxacin B. Urine analysis for bile pigments B. Ampicillin C. Feces analysis for stercobilin C. Cefazolinum D. Blood test for bilirubin D. Azithromycin E. Thymol turbidity test E. Chloramphenicol 17. An infant is 2.5 months old. The onset of 22. A 3-year-old girl is being treated at the disease was gradual, the child had normal a resuscitation unit with diagnosis ”acute body temperature but presented with slight kidney failure, oligoanuric stage”. ECG: high T cough. Within a week the cough intensified, wave, extended QRS complex, displacement especially at night; on the 12th day the child of S-T interval downwards below the isoline. developed cough fits occurring up to 20 times What electrolyte imbalance is it? per day and followed by vomiting. There was one instance of respiratory arrest. Make the A. Hyperkalemia diagnosis: B. Hypokalemia C. Hypocalcemia A. Pertussis D. Hypercalcemia B. Parainfluenza E. Hyperphosphatemia C. Congenital stridor D. Respiratory syncytial infection 23. Mother of an 8-year-old girl complains E. Adenovirus infection that the child is too short and has excessive body weight. Objectively: obesity with fat 18. A 14-year-old boy presents with moderate deposits on the torso and face (round moon- bronchial asthma in its exacerbation period. like face), acne, striae on the thighs and lower What drug should be prescribed to stop an abdomen, hirsutism. What hormone can cause acute attack of expiratory dyspnea? such symptoms, when in excess? A. Salbutamol A. Cortisol B. Cromolyn sodium (Cromoglicic acid) B. Thyroxin C. Dexamethasone C. Te s t o s t e r o n e D. Lasolvan (Ambroxol) D. Insulin E. Strophanthine (cardiac glycosides) E. Glucagon 19. An infant is 3 weeks old. Since birth there 24. A 9-year-old girl complains of fever up has been observed periodical vomiting within to 38.5oC, headache, inertness, weakness, a few minutes after feeding. The amount of loss of appetite, stomachache, and frequent vomitive masses does not exceed the volume painful urination. Provisional diagnosis of of previous feeding. The infant has age- acute pyelonephritis is made. Clinical urine appropriate body weight. What is the most analysis: specific gravity - 1016, no protein, likely cause of this symptom? leukocytes - 10-15 in the vision field. What investigation method can verify the diagnosis A. Pylorospasm of urinary tract infection? B. Esophageal chalasia C. Adrenogenital syndrome A. Bacteriological inoculation of urine D. Pyloristenosis B. Rehberg test (creatinine clearance test) E. Esophageal achalasia C. Zymnytsky test (density measurement of 20. A newborn with gestational age of 31 daily diuresis) weeks presents with hypotonia and depressed D. Complete blood count consciousness. Hematocrit is 35%, general E. Clinical urine analyses, dynamic testing cerebrospinal fluid analysis shows increased 25. A newborn has Apgar score of 9. When content of erythrocytes and protein, and should the infant be put to the breast? low glucose. These data correspond with the clinical presentation of: A. In the delivery room B. After 12 hours A. Intracranial hemorrhage C. After 2 hours B. Meningitis D. On the 2nd day C. Sepsis E. On the 3rd day D. Anemia E. Prenatal infection 26. A woman with blood group B(III) 21. From urine of a 14-year-old boy with Rh(+) gave birth to a full-term healthy boy. the exacerbation of secondary obstructive Examination on the 3rd day of the infant’s life pyelonephritis Pseudomonas aeruginosa was shows him to have icteric tint to his skin. The isolated with a titer of 1000000 microbes per child has no problems with suckling, sleep 1 ml. What antibiotic is the most advisable in is nondisturbed. The abdomen is soft, the this case? liver protrudes by 2 cm from under the costal margin. Complete blood count: hemoglobin - 200 g/L, erythrocytes - 5.5 · 1012/L, total Педiатричний профiль 29 bilirubin - 62 mcmol/L, indirect bilirubin - 52 mcmol/L. What condition can be suspected? A. Epidemic pemphigus of newborn B. Syphilitic pemphigus A. Physiologic jaundice C. Streptococcal impetigo B. Congenital hepatitis D. Vulgar impetigo C. Hemolytic disease of the newborn due to E. Atopic dermatitis Rh incompatibility D. Biliary atresia 31. A 1.5-month-old child on breastfeeding E. Hemolytic disease of the newborn due to presents from birth with daily vomiting, ABO incompatibility irregular liquid foamy feces, and flatulence, which are resistant to antibacterial and 27. A 3-day-old infant with hyperbilirubinemia probiotic therapy; no increase of body mass (428 mcmol/L) developed disturbances is observed. The child’s condition improved, manifesting as periodical excitation and when breastmilk was substituted. What convulsions against the background of pathology is it? inertness, hypotension, hypodynamia, and inhibition of unconditioned reflexes, A. Lactase deficiency convergent strabismus, rotational nystagmus, B. Intestinal lambliasis (Giardiasis) and setting-sun eye phenomenon. What is the C. Infectious enteritis most likely cause of such symptoms? D. Drug-induced enteritis E. Functional dyspepsia A. Bilirubin encephalopathy B. Craniocerebral injury 32. A 12-year-old girl after a case of C. Brain tumor respiratory infection developed dyspnea at D. Hydrocephalus rest, paleness of skin. Heart rate is 110/min., E. Infantile cerebral paralysis BP is 90/55 mm Hg. Heart sounds are muffled. Borders of relative heart dullness: right - the 28. The left hand of a newborn is extended in parasternal line, upper - the III rib, left - all its joints, stretched along the torso, and 1,0 cm outwards from the midclavicular line. pronated in the forearm. Active movements Make the provisional diagnosis: of the shoulder joint are retained. The hand is flattened, atrophied, cold to touch, hangs A. Infectious myocarditis passively. Grasping and Babkin’s reflexes B. Functional cardiopathy are absent at the affected side. Hemogram C. Somatoform autonomic dysfunction indicators are normal. Make the most likely D. Hypertrophic cardiomyopathy diagnosis: E. Exudative pericarditis A. Inferior distal obstetrical paralysis 33. A 5-year-old child has body temperature B. Osteomyelitis risen up to febrile numbers, suffers from C. Proximal obstetrical paralysis inertness, weakness. Examination revealed D. Complete obstetrical paralysis hemorrhage on the skin of limbs and E. Hypoxic-ischemic encephalopathy torso. Enlargement of cervical and axillary lymph nodes can be detected. The liver 29. A 15-year-old girl complains of dizziness is 4 cm below the costal arch; the spleen and sensation of lack of air that she develops is 6 cm below the costal arch. Blood test: in emotionally straining situations. Relief erythrocytes - 2.3 · 1012/L, Hb- 60 g/L, occurs after she takes corvalol. Objectively: platelets - 40 · 109/L, leukocytes - 32.8 · 109/L, hyperhidrosis and marble-like pattern of eosinophiles - 1%, band neutrophiles - 1%, the skin of her palms and feet. Clinical segmented neutrophiles - 12%, lymphocytes and instrumental examination revealed no - 46%, monocytes - 1%, blasts - 40%, organic alterations of the central nervous, Duke’s bleeding time test result is 9 min. cardiovascular, and respiratory systems. What What examination is necessary to make the provisional diagnosis can be made? diagnosis? A. Somatoform autonomic dysfunction A. Myelogram (bone marrow biopsy) B. Obstructive bronchitis B. Lymph nodes biopsy C. Bronchial asthma C. Abdominal US D. Stenosing laryngotracheitis D. Detection of hepatitis markers E. Acute epiglottitis E. Analysis of dynamic platelet function 30. 10 days after birth a newborn developed o 34. ECG revealed the following in a 10-year- sudden fever up to 38,1 C. Objectively: the old child: sharp acceleration of the heart rate skin in the region of navel, abdomen and - 240/min., P wave overlaps with T wave chest is erythematous; there are multiple and deforms it, moderate lengthening of PQ pea-sized blisters with no infiltration at the interval, QRS complex is without alterations. base; isolated bright red moist erosions with What pathology does this child have? epidermal fragments are observed on the periphery. What is the provisional diagnosis? Педiатричний профiль 30

A. Paroxysmal atrial tachycardia A. Left-sided diaphragm paresis B. Atrial hypertrophy B. Congenital pneumonia C. Ventricular hypertrophy C. Left-sided pneumothorax D. WPW syndrome D. Respiratory distress syndrome E. Extrasystole E. Transient tachypnea of the newborn 35. A 13-year-old girl complains of fatigability, 39. A 12-year-old child had three attacks frequent headaches, cardialgia. Eight years of acute rheumatic fever accompanied by ago she had a case of pyelonephritis. Urine carditis. Examination revealed the symptoms analyses periodically revealed leukocyturia. of chronic tonsillitis, mitral insufficiency, and The child has undergone no further treatment. carious teeth. What is the optimal method of On examination: increased BP up to 150/100 secondary prophylaxis? mm Hg. Ultrasound investigation revealed significant reduction of the right kidney. What A. Year-round bicillin prophylaxis until the age process is leading in arterial hypertension of 25 pathogenesis in this case? B. Course of cardiotrophic drugs twice a year C. Year-round bicillin prophylaxis for 3 years A. Hyperactivity of renin-angiotensin system D. Tonsillectomy B. Disturbance of water-electrolytic balance E. Oral cavity sanitation C. Disturbance of renal circulation D. Hypersympathicotonia 40. A 3-year-old child has been delivered to a E. Increased cortisol level hospital with complaints of pain in the legs, fever, loss of appetite. Objectively: pale skin 36. An 9-year-old child was hospitalized and mucosa, hemorrhagic rash. Lymph nodes for fever up to 39,8oC, inertness, moderate are enlarged, painless, dense and elastic, not headache, vomiting. Examination revealed matted together. Bones, joints, and abdomen meningeal symptoms. Lumbar puncture are painful. The liver and spleen are enlarged. was performed. The obtained fluid was Hemogram: Hb- 88 g/l, color index - 1,3, characterised by increased opening pressure, platelets - 80 · 109/l, leukocytes - 25, 8 · 109/l, was transparent, with the cell count of 450 lymphoblasts - 70%, ESR- 52 mm/hour. Make cells per 1 mcL (mainly lymphocytes - 90%), the provisional diagnosis: glucose level of 3,6 mmol/l. What agent could have caused the disease in the child? A. Acute leukemia B. Thrombocytopenic purpura A. Enterovirus C. Acute rheumatic fever B. Neisseria meningitidis D. Infectious mononucleosis C. Mycobacterium tuberculosis E. Hemorrhagic vasculitis (Henoch-Schonlein D. Staphylococcus aureus purpura) E. Streptococcus pneumoniae 41. During last several weeks an 11-year- 37. A 22-day-old infant developed old girl has been complaining of dyspnea subcutaneous red nodes from 1.0 to 1.5 cm in and edema of shins and feet after physical size on the scalp; later the nodes suppurated. exercise. After a long rest or sleep through the Temperature increased up to 37.7oC, night her edemas diminish significantly. On intoxication symptoms appeared, regional clinical examination there are enlarged liver lymph nodes enlarged. Complete blood count: and rasping systolic murmur over the cardiac anemia, leukocytosis, neutrocytosis, increased area. Blood and urine analyses are without ESR. What diagnosis will you make? changes. What is the most likely cause of the child’s edema? A. Pseudofurunculosis B. Pemphigus A. Heart failure C. Vesiculopustulosis B. Angioneurotic edema D. Scalp phlegmon C. Acute pyelonephritis E. - D. Hepatocirrhosis E. Nephrotic syndrome 38. A full-term newborn (born with the body weight of 3900 g at gestational age of 39 42. An 8-year-old girl with complaints weeks) on the first day of his life developed of painful urination, frequent low-volume respiratory disturbances: dyspnea, arrhythmic urination, and leukocyturia was diagnosed respiration, cyanosis attacks. On examination with acute cystitis. 10 days before the there is paradoxical respiration observed and disease onset she was treated by the left side of the chest lags behind in the act of gynecologist for acute vulvitis. 5 days ago breathing. On auscultation the respiration is she presented with mild catarrhal symptoms. weakened in the lungs on the left. Neurologist Her mother ascribes the child’s disease to her diagnosed the patient with left-sided Erb- overexposure to cold. Specify the most likely Duchenne palsy.Complete blood count shows infection route: no changes. What is the most likely diagnosis? Педiатричний профiль 31

A. Ascending between doses of vaccine in this case? B. Descending C. Hematogenic A. 1 month D. Contact B. 6 months E. Lymphogenic C. 2 months D. 3 months 43. A 5-year-old boy complains of severe E. 12 months dyspnea and sensation of lack of air. Objectively the child assumes orthopneic 46. A 6-month-old infant is not vaccinated. position, presents with marked peripheral The physician recommends a DPT cyanosis, drumstick fingers, nail plates (diphtheria, pertussis, tetanus) vaccination resemblinga”clockface”, the borders of but the mother is absolutely against this cardiac dullness are bilaterally extended; procedure. Choose the most substantial coarse systolic murmur can be detected over argument in favor of vaccination: the whole surface of the heart and is especially pronounced in the II intercostal area on the A. Risk of lethal consequences left near the sternum. What disease can be B. Epidemic risk for the others characterized by such presentations? C. Personal professional experience D. High quality of vaccines A. Fallot’s tetrad E. - B. Dilated cardiomyopathy C. Defect of the interventricular septum 47. During administration of planned DPT D. Primary bacterial endocarditis vaccination the child suddenly developed E. Defect of the interatrial septum acute anxiety, signs of pain response, dyspnea, grunting respirations, cutis marmorata, cold 44. A 10-year-old boy is delivered into a sweat. Objectively the child’s consciousness polytrauma unit after he received a blunt is disturbed, heart rate is 150/min., blood trauma of the thorax, having fallen from pressure is 60/40 mm Hg, heart sounds the bicycle. Upon hospitalization his blood are muffled. The child was diagnosed with pressure is 110/80 mm Hg, heart rate is anaphylactic shock. What drug should be 96/min. Chest X-ray is noncontributive to the administered first? diagnosis. Echocardiogram shows free liquid in the pericardial cavity, in the amount of up A. Epinephrine to 100 ml. In an hour after the hospitalization B. Lasix (Furosemide) the patient started to develop increasing signs C. Suprastin (Chloropyramine) of heart failure: jugular venous distention, D. Euphylline (Aminophylline) decreased blood pressure down to 90/70 E. Analgin (Metamizole) mm Hg, tachycardia up to 120/min. On auscultation muffled heart sounds. What 48. Among first-year schoolchildren there was would be the primary tactics of a physician? a case of measles registered. A 7-year-old boy from the same group was not vaccinated A. Pericardiocentesis (pericardial puncture) against measles due to refusal of his parents. B. Cardiac glycosides intravenously His clinical history has no cases of measles C. Constant oxygenotherapy in the past and is not contraindicatory to D. Diuretics intravenously immunobiological agents. Choose the most E. Antibiotics intravenously rational tactics of measles prevention in this schoolboy: 45. A healthy child 1 year and 5 months of age is being vaccinated against hepatitis B. A. Measles-Mumps-Rubella vaccine The child did not receive the first dose of B. Isolation for 20 days the vaccine previously, while in the maternity C. Antiviral agents hospital. The doctor makes an individual D. Antibiotics vaccination schedule for this child and plans E. Immunomodulators the administration of the next dose of the vaccine. What is the minimum interval Акушерство i гiнекологiя 32

1. A 28-year-old woman has been weight of the parturient woman is 62 delivered to a hospital with acute pain in the kg. Fetus is in a longitudinal lie, the lower abdomen. There was a brief syncope. head is engaged to the pelvic inlet. Belly The delay of menstruation is 2 months. circumference is 100 cm. Uterine fundus Objectively: the patient has pale skin, BP- height is 35 cm. What body weight of the 90/50 mm Hg, Ps- 110/min. Lower abdomen fetus can be expected? is extremely painful. Vaginal examination reveals uterus enlargement. Promtov’s A. 3.5 kg sign (pain during bimanual gynecological B. 4kg examination) is positive. Right uterine C. 2.5 kg appendages are enlarged and very painful. D. 3kg Posterior vault hangs over. What is the most E. 4.5 kg likely diagnosis? 6. A 22-year-old woman, gravida 1, para 0 A. Right-sided tubal pregnancy arrived with complaints of sharply painful B. Right ovary apoplexy contractions that occur every 4-5 minutes C. Acute right-sided salpingoophoritis and last for 25-30 seconds. Amniotic fluid did D. Pelvioperitonitis not burst. The fetus is in transverse lie, fetal E. Incipient abortion heartbeats are not affected. Contraction ring is acutely painful, located obliquely at the 2. A parturient woman complains of pain in umbilicus. What is the most likely diagnosis? her mammary gland. In the painful area there is an infiltration 3x4 cm in size with softened A. Impending uterine rupture center. Body temperature is 38,5oC.Whatis B. Uterine tetany the most likely diagnosis? C. Excessive uterine activity during labor D. Discoordinated labor A. Acute suppurative mastitis E. Uterine rupture B. Pneumonia C. Pleurisy 7. A 30-year-old multigravida has been D. Milk retention in labour for 18 hours. 2 hours ago the E. Birth trauma pushing stage began. Fetal heart rate is clear, rhythmic, 136/min. Vaginal examination 3. A 29-year-old woman came to a reveals complete cervical dilatation, the fetal gynecologist with complaints of irritability, head in the pelvic outlet plane. Sagittal tearfulness, headache, nausea, occasional suture is in line with obstetric conjugate, vomiting, pain in the heart area, tachycardia the occipital fontanel is near the pubis. The attacks, memory impairment, meteorism. patient has been diagnosed with primary These signs appear 6 days before uterine inertia. What is the further tactics menstruation and disappear the day before of labour management? menstruation or during its first 2 days. On vaginal examination: the uterus and uterine A. Outlet forceps appendages are without alterations. What B. Labour stimulation diagnosis is the most likely? C. Cesarean section D. Skin-head Ivanov’s forceps A. Premenstrual syndrome E. Vacuum extraction of the fetus B. Algodismenorrhea C. Ovarian apoplexy 8. A 20-year-old woman on the 10th day after her discharge from the maternity ward D. Genital endometriosis o E. Neurosis developed fever up to 39 C and pain in her left mammary gland. On examination the 4. A primigravida at the term of 20 weeks mammary gland is enlarged, in its upper complains of pain in her lower abdomen, outer quadrant there is a hyperemic area. In smearing blood-streaked discharge from the this area a dense spot with blurred margins genital tracts. Uterine tone is increased, can be palpated. The patient presents with fetus is mobile. On vaginal examination: the lactostasis and no fluctuation. Lymph nodes uterus is enlarged according to the term, in the right axillary crease are enlarged and uterine cervix is shortened to 0,5 cm, external painful. Specify the correct diagnosis: cervical orifice is open by 2 cm. What is the most likely diagnosis? A. Lactational mastitis B. Abscess A. Risk of late abortion with hemorrhage C. Erysipelas B. Risk of late abortion without hemorrhage D. Dermatitis C. The process of late abortion E. Tu m o r D. Incomplete late abortion E. Attempted late abortion 9. A 16-year-old girl has primary amenorrhea, no pubic hair growth, normally 5. The pregnancy is full term. The body developed mammary glands; her genotype is Акушерство i гiнекологiя 33

46 ХY; uterus and vagina are absent. What is appendages can be detected. What is the your diagnosis? most likely diagnosis? A. Testicular feminization syndrome A. Sheehan’s syndrome (postpartum B. Mayer-Rokitansky-Kuster-Hauser hypopituitarism) syndrome B. Ovarian amenorrhea C. Cushing syndrome C. Turner’s syndrome D. Sheehan syndrome D. Ovarian exhaustion syndrome E. Cushing disease E. Galactorrhea-amenorrhea syndrome 10. 6 hours ago the waters of a 30-year- 14. A 28-year-old woman complains of old gravida 1, para 0, burst; her preliminary increased intervals between menstruations, period was pathologic and lasted for over 2 up to 2 months, and hirsutism. Gynecological days; the term of pregnancy is 39 weeks. No examination revealed the following: ovaries labor activity is observed. Fetal head presents are enlarged, painless, and dense; no above the pelvic inlet. Fetal heartbeats are alterations of the uterus. US of the lesser 142/min., clear and rhytmic. On vaginal pelvis: ovaries are 4-5 cm in diameter, with examination the uterine cervix is not dilated. numerous enlarged follicles on the periphery. What further tactics should the doctor X-ray of the skull base: sellar region is choose? widened. What is the most likely diagnosis? A. Perform cesarean section A. Stein-Leventhal syndrome (polycystic B. Induce cervical dilation with prostaglandins ovarian syndrome) C. Stimulate the labor with oxytocin B. Algodismenorrhea D. Wait for the onset of spontaneous labor C. Sheehan syndrome (postpartum E. Prolong the pregnancy, while providing hypopituitarism) antibacterial treatment D. Premenstrual syndrome 11. A 23-year-old woman came the the E. Morgagni-Stewart-Morel syndrome gynecologist with complaints of blood smears (metabolic craniopathy) from her genital tracts that have been observed for a long time. Her menstruation 15. The gynecology unit received a patient has been delayed for 8 weeks. Examination with uterine bleeding that started 6 hours shows the uterine body to be enlarged up after induced abortion at the term of 11-12 to 14 weeks of pregnancy. US detected weeks. Objectively the skin is pale, pulse a vesicular mole. What tactics should the is 100/min., blood pressure is 100/70 mm doctor choose? Hg. On vaginal examination the uterus is painless, its enlargement corresponds to the A. Curettage of the uterine cavity 10th week of pregnancy; uterine cervix is B. Hormonal treatment dilated enough to let in one finger, there C. Hemostatic treatment are fragments of the fertilized ovum. What D. Supravaginal uterine amputation actions should be taken next: E. Uterectomy A. Urgent repeated curettage of the uterine 12. A 46-year-old woman came to a maternity cavity clinic with complaints of moderate blood B. Uterotonic drugs discharge from the vagina, which developed C. Treatment for acute anemia after the menstruation delay of 1,5 months. D. Antibacterial agents On vaginal examination: the cervix is clean; E. Prescribe rest and continue to monitor the the uterus is not enlarged, mobile, painless; patient’s condition appendages without changes. Make the diagnosis: 16. A woman came to the general practitioner with complaints of fatigability, A. Dysfunctional uterine bleeding significant weight loss, weakness, and loss B. Adenomyosis of appetite. She has been presenting with C. Ectopic pregnancy amenorrhea for the last 8 month. One year D. Submucous uterine myoma ago she gave birth to a live full-term child. E. Cancer of the uterine body Blood loss during delivery was 2 liters. The woman received blood transfusion and 13. A 30-year-old woman complains of blood components. What is the most likely amenorrhea that lasts for 2 years after she diagnosis? has given birth, loss of hair and body weight. The labor was complicated with hemorrhage caused by uterine hypotonia. Objectively the patient is of asthenic type, her external genitalia are hypoplastic, the uterine body is small in size and painless. No uterine Акушерство i гiнекологiя 34

A. Sheehan’s syndrome (postpartum A. Juvenile uterine hemorrhage hypopituitarism) (Dysfunctional) B. Stein-Leventhal syndrome (polycystic B. Hypomenstrual syndrome ovary) C. Inflammation of uterine appendages C. Turner’s syndrome (Pelvic inflammatory disease) D. Homologous blood syndrome D. Pelviperitonitis E. Somatoform autonomic dysfunction E. Endometritis 17. A 22-year-old woman complains of 21. A 15-year-old adolescent girl came itching and profuse discharge from her the the gynecologist with complaints of genital tracts. The condition developed painful menstruations that are accompanied 10 days ago after a sexual contact. by nausea, vomiting, and dizziness. Her Bacterioscopy of a discharge sample menarche was at 12. Menstruations became detected trichomonads. What drug should painful since she was 14, remain regular. be prescribed for treatment in this case? What treatment should be prescribed in this case? A. Metronidazole B. Ampicillin A. Analgesics, antispasmodics, C. Erythromycin antiprostaglandine therapy D. Zovirax (Acyclovir) B. Antiinflammatory treatment only E. Valcyclovir C. Antihemorrhagic agents D. Antiandrogen therapy 18. A multigravida on the 38th week of her E. Vitamin supplements pregnancy complains of increased BP up to 140/90 mm Hg, edema of the shins for 2 22. A 20-year-old woman, gravida 2, para 1 weeks. In the last month she gained 3.5 kg has been in labor for 4 hours. Her condition is of weight. Urine analysis: protein - 0.033 g/L. satisfactory. Moderately painful contractions Make the diagnosis: occur every 3 minutes and last for 35-40 seconds. The waters have not burst yet. A. Mild preeclampsia The fetus is in longitudinal position. Fetal B. Moderate preeclampsia heartbeats are 136/min., clear and rhytmic. C. Pregnancy hypertension Major segment of the fetal head is engaged to D. Severe preeclampsia the pelvic inlet. Vaginal examination shows E. Pregnancy edema smooth cervix of 6cm, amniotic sac is intact, sagittal suture is in the left oblique diameter, 19. A 37-year-old woman complains of acute occipital fontanel is on the right near the pain in the genital area, swelling of the symphysis pubis. What stage of the labor is labia, pain when walking. Objectively: body it? temperature is 38,7oC, Ps- 98/min. In the interior of the right labia there is a dense, A. Active phase of the first stage of normal painful tumor-like formation 5,0x4,5 cm in labor size, the skin and mucous membrane of B. Latent phase of the first stage of normal genitals are hyperemic, there is profuse foul- labor smelling discharge. What is the most likely C. The second stage of normal labor diagnosis? D. Precursors of childbirth E. Preliminary stage A. Acute bartholinitis B. Labial furuncle 23. A multigravida at 39 weeks of gestation C. Acute vulvovaginitis presenting with regular labour activity for 8 D. Bartholin gland cyst hours has been delivered to a hospital; the E. Carcinoma of vulva waters broke an hour ago. She complains of headache, seeing spots. BP is 180/100 20. A 14-year-old girl has been delivered to mm Hg. Urine test results: protein - 3,3 a gynecological department with complaints g/l, hyaline cylinders. Fetal heart rate is of profuse blood discharge from her genital 140/min, rhythmical. Vaginal examination tract for 2 weeks. Anamnesis: menstruation reveals complete cervical dilatation, the fetal since 13, irregular, painful, profuse; the last head is on the pelvic floor, sagittal suture is in one was 2 months ago. Objectively: pale line with obstetric conjugate, the occipital skin and mucosa, BP- 100/60 mm Hg, Hb- fontanel is under the pubis. What is the 108 g/l. The abdomen is soft and painless optimal tactics of labour management? on palpation. Rectal examination revealed no pathologies of reproductive organs. What A. Outlet forceps condition is it? B. Cavity forceps C. Cesarean section D. Vacuum extraction of the fetus E. Conservative labour management Акушерство i гiнекологiя 35

24. An 18-year-old woman complains of pain A. Pituitary tumour in her lower abdomen, profuse purulent B. Lactational amenorrhea discharge from the vagina, temperature C. Stein-Leventhal syndrome (polycystic rise up to 37,8oC. Anamnesis states that ovary syndrome) she had a random sexual contact the D. Sheehan’s syndrome (postpartum day before the signs appeared. She was hypopituitarism) diagnosed with acute bilateral adnexitis. E. Cushing’s disease On additional examination: leukocytes are present throughout all vision field, bacteria, 28. A 30-year-old woman complains of diplococci with intracellular and extracellular milk discharge from her breasts and no position. What is the most likely agent in the menstruation for the last 5 months. One given case? physiologic childbirth was 4 years ago. There are no maldevelopments of mammary A. Neisseria gonorrhoeae glands. Bimanual examination revealed B. Escherichia coli diminished uterus and normal sized ovaries. C. Chlamydia trachomatis MRI-scan shows no brain pathologies. D. Trichomona vaginalis Thyroid-stimulating hormone is within E. Staphylococcus aureus normal limits. Serum prolactin is high. What is the most likely diagnosis? 25. It is the 3rd day after the normal term labor; the infant is rooming-in with the A. Hyperprolactinemia mother and is on breastfeeding. Objectively: B. Hypothyroidism the mother’s general condition is satisfactory. C. Polycystic ovaries Temperature is 36.4oC, heart rate is 80/min., D. Pituitary adenoma BP is 120/80 mm Hg. Mammary glands E. Sheehan’s syndrome (postpartum are soft and painless; lactation is moderate, hypopituitarism) unrestricted milk flow. The uterus is dense, the uterine fundus is located by 3 fingers 29. A 25-year-old woman during self- width below the navel. Lochia are sanguino- examination detected a tumor in the upper serous, moderate in volume. Assess the external quadrant of her right mammary dynamics of uterine involution: gland. On palpation: painless, dense, mobile growth 2 cm in diameter is detected in A. Physiological involution the mammary gland; no changes in the B. Subinvolution peripheral lymph nodes are observed. On C. Lochiometra US of the mammary glands: in the upper D. Pathologic involution external quadrant of the right mammary E. Hematometra gland there is a space-occupying lesion of increased echogenicity 21х18 mm in size. The 26. A 6-year-old girl came to a general most likely diagnosis is: practitioner with her mother. The child complains of burning pain and itching in A. Fibrous adenoma her external genitalia. The girl was taking B. Breast cyst antibiotics the day before due to her suffering C. Diffuse mastopathy from acute bronchitis. On examination: D. Breast cancer external genitalia are swollen, hyperemic, E. Mastitis there is white deposit accumulated in the folds. The most likely diagnosis is: 30. A 45-year-old woman came to the maternity clinic with complaints of periodical A. Candidal vulvovaginitis pains in her mammary glands that start B. Trichomoniasis 1 day before menstruation and stop after C. Nonspecificvulvitis the menstruation begins. Palpation of the D. Helminthic invasion mammary glands detects diffuse nodes E. Herpes vulvitis predominantly in the upper outer quadrants. What is the most likely diagnosis? 27. A 22-year-old woman complains of amenorrhea for 8 months. Anamnesis A. Fibrocystic mastopathy states that menarche occured at the age of B. Breast cancer 12,5. Since the age of 18 the patient has C. Mastitis a history of irregular menstruation. The D. Hyperprolactinemia patient is nulligravida. The mammary glands E. Breast cyst are developed properly, nipples discharge drops of milk when pressed. Hormone test: 31. A woman complains of temperature prolactin level is 2 times higher than normal. increaseupto39oC, sharp pains in her lower CT reveals a bulky formation with diameter abdomen, and sanguinopurulent discharge of 4 mm in the region of sella. What is the from her genital tracts. From her case history most likely diagnosis? it is known that 6 days ago she underwent illegal abortion. Objectively her blood Акушерство i гiнекологiя 36 pressure is 100/60 mm Hg, pulse is 110/min. A. Early delivery Abdominal rigidity, rebound tenderness B. Delivery at 37 weeks of gestation (Bloomberg’s sign), and painful palpation C. Screening for Rh-antibodies in 2 weeks of the lower abdomen are observed. On and urgent delivery in case of further increase bimanual examination the uterus is enlarged of antibody titer up to 7 weeks of pregnancy, painful, and soft; D. Introduction of anti-Rh (D) posterior vaginal fornix overhangs. Make the immunoglobulin diagnosis: E. Ultrasound for signs of hemolytic disease of the fetus A. Pelviperitonitis B. Endometritis 35. A 55-year-old woman came to a C. Acute adnexitis gynecologist with complaints of leukorrhea D. Pyosalpinx and bloody discharge from the vagina after E. Metroendometritis 5 years of menopause. Anamnesis states no pregnancies. Bimanual examination: the 32. A 14-year-old girl came to a general uterus and uterine appendages are without practitioner with complaints of weakness, changes. During diagnostic curettage of the loss of appetite, headache, rapid fatigability. uterine cavity the physician scraped off Her last menstruation was profuse and enchephaloid matter. What is the most likely lasted for 14 days after previous delay of 2 diagnosis in this case? months. Objectively: the skin is pale, heart rate is 90/min., BP is 110/70 mm Hg, Hb A. Endometrial carcinoma is 88 g/l. Rectal examination: the uterus B. Adenomyosis and its appendages are without changes, C. Subserous uterine myoma no discharge from the genital tracts. What D. Cervical carcinoma complication occurred in the patient? E. Ovarian carcinoma A. Posthemorrhagic anemia 36. A 24-year-old pregnant woman on her B. Somatoform autonomic dysfunction of 37th week of pregnancy has been delivered to hypotonic type a maternity obstetric service with complaints C. Migraine of weak fetal movements. Fetal heartbeats D. Gastritis are 95/min. On vaginal examination the E. Dysmenorrhea uterine cervix is tilted backwards, 2 cm long, external orifice allows inserting a 33. A postpartum woman on the 12th day fingertip. Biophysical profile of the fetus after the normal delivery complains of pain equals 4 points. What tactics of pregnancy localized in her left gastrocnemius muscle. management should be chosen? Body temperature is 37,2oC; pulse is 85/min, rhythmic; blood pressure is 128/80 mm Hg. A. Urgent delivery via cesarean section Mammary glands are soft and painless. The B. Treatment of placental dysfunction and uterus is behind the pubis. The left leg in repeated analysis of the fetal biophysical the area of gastrocnemius muscle is by 3 profile on the next day cm larger than the right leg in the diameter. C. Doppler measurement of blood velocity in Internal organs present no pathologies. What the umbilical artery complication can be suspected? D. Urgent preparation of the uterine cervix for delivery A. Deep vein thrombosis of the shin E. Treatment of fetal distress, if ineffective, B. Iliofemoral thrombosis then elective cesarean section on the next day C. Varicose veins of lower extremities D. Endometritis 37. During regular preventive gynecological E. Myositis examination a 30-year-old woman was detected to have dark blue punctulated 34. Examination of a Rh-negative pregnant ”perforations” on the vaginal portion of woman at 32 weeks of gestation revealed a the uterine cervix. The doctor suspects four-time rise of Rh-antibody titer within endometriosis of the vaginal portion of the 2 last weeks; the titer is 1:64. The first two uterine cervix. What investigation method pregnancies resulted in antenatal fetal death would be most informative for diagnosis due to hemolytic disease. What is the optimal confirmation? tactics of pregnancy management? A. Colposcopy, target biopsy of the cervix B. US of the lesser pelvis C. Hysteroscopy D. Curettage of the uterine cavity E. Hormone testing 38. A 26-year-old woman came to a gynecologist for a regular check-up. She has Акушерство i гiнекологiя 37 no complaints. Per vaginum: the uterus lies second pregnancy. She complains of fever, in anteflexion, not enlarged, dense, mobile, chills, nausea, vomiting, lumbar pain, and painless. On the left from the uterus in dysuria. Costovertebral angle tenderness is the area of uterine appendages there is present on both sides. Urine analysis: pyuria, a mobile painless outgrowth that can be bacteriuria. Blood test: leukocytosis. What is moved independently from the uterus. On the most likely diagnosis? the right the appendages cannot be detected. What additional investigation would be A. Gestational pyelonephritis informative for diagnosis clarification? B. Cystitis C. Pyelitis A. Ultrasound of the lesser pelvis D. Glomerulonephritis B. Metrosalpingography E. Latent bacteriuria C. Examination for urogenital infection D. Colposcopy 42. A 32-year-old pregnant woman at the E. Colonoscopy term of 5-6 weeks was vaccinated against influenza along with her whole family. At 39. A postparturient woman, who has that time she was not aware of her pregnancy. been breastfeeding for 3 weeks, made The pregnancy is wanted. The woman needs an appointment with the doctor. For the an advice from the family doctor regarding last 6 days she has been feeling unwell, the maintenance of her pregnancy, namely complains of body temperature of 38- whether there is a risk of fetal malformations 39oC, general weakness; within the last because of received vaccination. What advice 2 days she developed pain and redness should the doctor give in this case? in the area of her right mammary gland. Examination revealed her mammary gland A. Vaccination against influenza is safe during to be significantly enlarged and deformed; pregnancy breast tissue fluctuations and lymphadenitis B. Therapeutic abortion is recommended are observed. What type of mastitis is the C. Immediate ultrasound of the lesser pelvis most likely? is necessary D. Test for antibodies against influenza virus A. Phlegmonous mastitis is necessary B. Serous mastitis E. An infectious diseases specialist must be C. Infiltrative mastitis consulted D. Lactostasis E. Mammary edema 43. A 17-year-old girl has made an appointment with the doctor. She plans to 40. On the 9th day after childbirth the begin her sex life. No signs of gynecological obstetric patient developed high fever up pathology were detected. In the family to 38oC. She complains of pain in the right history there was a case of cervical cancer mammary gland. The examination revealed that occurred to the patient’s grandmother. the following: a sharply painful infiltrate can The patient was consulted about the be palpated in the right mammary gland, the maintenance of her reproductive health. skin over the infiltrate is red, subareolar area What recommendation will be the most and nipple are swollen and painful. What is helpful for prevention of invasive cervical your diagnosis? cancer? A. Abscess of the right mammary gland A. Vaccination against human papillomavirus B. Mastopathy (HPV) C. Cancer of the right mammary gland B. Vitamins, calcium, omega-3 D. Serous mastitis C. Immunomodulators E. Fibrous cystic degeneration of the right D. Antiviral and antibacterial drugs mammary gland E. Timely treatment of sexually transmitted diseases 41. A woman is on the 32nd week of her Гiгiєна та органiзацiя охорони здоров’я 38

1. A regional cardiologist is given a task The complex of symptoms indicates the to develop a plan for preventive measures initial stage of chronic intoxication with: aimed at decreasing cardiovascular mortality rates. What measures should be A. Lead planned for secondary prevention? B. Manganese C. Mercury A. Prevention of recurrences and D. Tin complications E. Ethanol B. Referring patients for sanatorium-and-spa treatment 6. A 10-year-old girl exhibits high level

C. Prevention of diseases of physical development ( +3σ), her D. Referring patients for inpatient treatment body length increased by 10 cm within a E. Optimization of lifestyle and living year (which is double the norm for her conditions age group), the number of permanent teeth corresponds with the age norm (20), 2. Caries morbidity rate is 89% among the development of her secondary sex residents of a community. It is determined characteristics is three years ahead of her that fluorine content in water is 0.1 mg/L. age (Ма, Р, Ах, Menarche). Development What preventive measures should be taken? rate ahead of her biological age can occur A. Water fluorination due to: B. Tooth brushing A. Endocrine disorders C. Fluorine inhalations B. Acceleration D. Sealant application C. Certain components of her diet E. Introduce more vegetables to the diet D. Sports training 3. A 39-year-old man, a battery attendant, E. Deficient hygienic education suddenly developed weakness, loss of 7. Examination of a group of persons appetite, nonlocalized colicky abdominal living on the same territory revealed pains, and nausea. Objectively his skin the following common symptoms: dark- is gray; there is pink-gray stripe on his yellow pigmentation of the tooth enamel, gums; the stomach is soft and sharply diffuse osteoporosis of bone apparatus, painful. Blood test detected erythrocytes ossification of ligaments and joints, with basophilic stippling and anemia. functional disorders of the central nervous The patient has a history of peptic ulcer system. This condition may be caused by disease of the stomach. There is tendency the excessive concentration of the following to constipation. What is the most likely microelement in food or drinking water: provisional diagnosis? A. Fluorine A. Saturnism (lead poisosning) B. Copper B. Acute appendicitis C. Nickel C. Perforation of gastric ulcer D. Iodine D. Acute cholecystitis E. Cesium E. Chronic alcoholism 8. A worker of a blowing shop complains 4. A 9-month-old infant presents with of headache, irritability, sight impairment delayed tooth eruption and fontanel closure, - he sees everything as if through a ”net”. weakness, and excessive sweating. What Objectively: hyperemic sclera, thickened type of hypovitaminosis is the most likely cornea, decreased opacity of pupils, visual in this child? acuity is 0.8 in the left eye, 0.7 in the A. Hypovitaminosis D right eye. The worker uses no means of B. Hypovitaminosis C personal protection. What diagnosis is the most likely? C. Hypovitaminosis B1 D. Hypovitaminosis B6 A. Cataract E. Hypovitaminosis A B. Conjunctivitis 5. During the periodic medical examination C. Keratitis an assembly fitter (works on soldering D. Blepharospasm E. Progressive myopia details) didn’t report any health problems. Closer examination revealed signs of 9. The inpatient surgery unit has introduced asthenic-vegetative syndrome. Blood the method of laparoscopic cholecystectomy included red blood cells with basophilic into its practice. As the result the average aggregations and a somewhat higher duration of postoperative care provided number of reticulocytes, urine had a high to the patients could be reduced to concentration of delta-aminolevulinic acid. 3.4±0.8 days compared to 7.3±1.1 days Гiгiєна та органiзацiя охорони здоров’я 39 that were required after non-laparoscopic A. Lead and lead salts cholecystectomy. What method of medical B. Tin statistics can confirm the statistical C. Carbon monoxide significance of the difference between these D. Nitric oxide two estimates? E. Zinc A. Calculation of Student’s confidence 13. Due to introduction of a new treatment interval method, average duration of therapy in B. Calculation of correlation coefficient the experimental group was 12.3±0.2 days C. Calculation of standardized ratio compared to 15.4±0.4 days in the control D. Calculation of average values (measures group that was treated by the old method. of central tendency) What calculations should be made to E. Calculation of relative values estimate the statistical significance of the difference in the results? 10. In April during the medical examination of various population groups, 27% of A. T-test (Student’s t-distribution) individuals presented with low working B. Sign test (Z-test) ability and rapid fatigability. The following C. Matching factor (chi-squared test) symptoms were observed in the affected D. Wilcoxon T-test individuals: swollen friable gingiva that E. Kolmogorov-Smirnov test bleeds when pressed, hyperkeratosis follicularis not accompanied by skin dryness. 14. A patient, who had eaten canned These symptoms most likely result from the mushrooms (honey agaric) three days ago, following pathology: developed vision impairment (diplopia, mydriasis), speech disorder, disturbed A. C-hypovitaminosis swallowing. What type of food poisoning B. Parodontosis occurred in the patient? C. A-hypovitaminosis

D. þ1-hypovitaminosis A. Botulism E. Polyhypovitaminosis B. Food toxicoinfection C. Fusariotoxicosis 11. During regular check-up the doctor D. Honey agaric poisonong examines a young woman, a student, with E. Lead salts poisoning height of 162 cm and weight of 59 kg. She complains that in the evening she becomes 15. Establishments participating in unable to see clearly the objects around her. medical examinations include: medical and Objectively her skin is dry, presents with preventive treatment facilities, hygiene and hyperkeratosis. Her daily ration has the preventive treatment facilities, sociomedical following vitamin content: vitamin A -0.5 expert committees, Ministry of Defence mg, vitamin B1 - 2.0 mg, vitamin B2 -2.5mg, medical committees, Ministry of Domestic vitamin B6 - 2 mg, vitamin C -70mg.Make Affairs medical committees, forensic the diagnosis: medicine agency, etc. Specify what service deals with sociomedical assessment of A. A-hypovitaminosis temporary disability: B. B1-hypovitaminosis C. B2-hypovitaminosis A. Medical and preventive treatment D. B6-hypovitaminosis facilities E. C-hypovitaminosis B. Hygiene and preventive treatment facilities 12. A man works in casting of nonferrous C. Sociomedical expert committees metals and alloys for 12 years. In the D. Ministry of Defence medical committees air of working area there was registered E. Ministry of Domestic Affairs medical high content of heavy metals, carbon committees monoxide, and nitrogen. During periodic health examination the patient presents with 16. During assessment of work conditions asthenovegetative syndrome, sharp pains at the factory manufacturing mercury in the stomach, constipations, pain in the thermometers, the content of mercury hepatic area. In urine: aminolevulinic acid vapors in the air of the working area and coproporphyrin are detected. In blood: is revealed to exceed the maximum reticulocytosis, low hemoglobin level. Such concentration limit. Specify the main intoxication is caused by: pathway of human body exposure to mercury: Гiгiєна та органiзацiя охорони здоров’я 40

A. Respiratory organs test results: Hb- 90 g/l, anisocytosis, B. Intact skin poikilocytosis. The most likely causative C. Damaged skin factor of this condition is the inadequate D. Gastrointestinal tract intake of: E. Mucous tunics A. Iron 17. During health assessment of car drivers B. Copper and police officers on point duty, the C. Zinc physicians detected carboxyhemoglobin in D. Magnesium the blood of the patients, weakened reflex E. Selene responses, disturbed activity of a number of enzymes. Revealed professional health 22. To assess the effectiveness of medical disorders are most likely to be associated technologies and determine the power and with the effect of: direction of their effect on the public health indicators, the research was conducted to A. Carbon monoxide study the immunization rate of children and B. Sulfurous anhydride measles incidence rate by district. What C. Mental stress method of statistical analysis should be D. Aromatic hydrocarbons applied in this case? E. Nitric oxide A. Calculation of correlation coefficient 18. In the process of hiring, a prospective B. Calculation of morbidity index among the employee has undergone preventive nonvaccinated medical examination and was declared C. Calculation of coefficient of agreement fit to work in this manufacturing D. Calculation of standardized ratio environment. What type of preventive E. Calculation of statistical significance of medical examination was it? the difference between two estimates A. Preliminary 23. Posture of an 11-year-old boy was B. Scheduled determined during preventive examination. C. Periodical The child presents with curled forward D. Specific rounded shoulders, the head is bowed E. Comprehensive forward, the thorax is flattened, the stomach is bulging. In the vertebral column there 19. An employee has been sick for 4 months, are deepened cervical and lumbar flexures. further treatment is necessary, the patient What posture does the child have? is unable to work. Who is authorized to provide further disability examination of A. Kyphosis this patient? B. Lordosis C. Stooping A. Sociomedical expert committee D. Corrected B. Medical consultative board E. Normal C. Physician in charge and the head of the department 24. The objective of a statistical research was D. Chief physician of a medical facility to find out to what extent the population E. Deputy chief responsible for disability peruses the available medical services. For examination this purpose 300 residents of the area were interviewed. Information was collected by 20. On laboratory investigation of a pork means of a special questionnaire. What sample there is 1 dead trichinella detected method of collecting information was used in 24 sections. This meat should be: by the researchers? A. Handed over for technical disposal A. Anamnestic (history-taking) B. Allowed for sale with no restrictions B. Immediate registration C. Processed and sold through public catering C. Immediate examination network D. Doing extracts D. Processed for boiled sausage production E. - E. Frozen until the temperature of - 10oC is reached in the deep layers, with subsequent 25. A 47-year-old man is employed at exposure to cold for 15 days the weaving workshop, has 15-year-long record of service at this factory; his 21. Examination of a 43-year-old man work conditions are associated with high- objectively revealed pallor of skin and frequency and high-intensity noise. During mucous membranes, loss of tongue papillae, periodical examination he was diagnosed transverse striation of fingernails, cracks with occupational deafness. What are the in the mouth corners, tachycardia. Blood grounds for making such a diagnosis? Гiгiєна та органiзацiя охорони здоров’я 41

A. Audiometry data and hygienic assessment A. Primary healthcare of working environment B. Emergency aid B. Record of service at this factory C. Secondary healthcare C. Noise characteristic at this factory D. Tertiary healthcare D. Central nervous system examination E. Palliative care results E. Inner ear examination results 30. 40-50 minutes after the completion of repair works conducted in a closed 26. Estimation of community health level garage, with car engine running, the repair involved analysis of a report on diseases workers developed severe headache in the registered among the population of district temporal area, nausea, tinnitus, vertigo, etc. under charge (reporting form 12). What These symptoms are characteristic of acute index is calculated based on this report? poisoning with: A. Prevalence A. Carbon monoxide B. Index of pathological affection B. Aldehydes C. Index of morbidity with temporary C. Organochlorides disability D. Hydrogen sulfide D. Index of hospitalized morbidity E. Fluoride E. Index of basic non-epidemic morbidity 31. A 45-year-old veterinary worker has 27. A 60-year-old man has a diet consisting made an appointment with the doctor of unvaried food staples: mostly cereals, for regular examination. In his duties he potato, pasta; few vegetables and little fats frequently deals with animals, however (especially animal fats). During medical he denies working with rabies-affected examination he complains of deterioration animals. Previously he has received no of his twilight vision. This condition can be antirabic vaccination. What should the caused by lack of: doctor recommend in this case? A. Retinol A. Preventive immunization with antirabic B. Amino acids vaccine C. Fats B. Vaccination in case of contact with sick D. Calcium animal E. Carbohydrates C. Preventive immunization with rabies immunoglobulin 28. A 52 year old man came to see his D. Administration of antirabic vaccine and family physician complaining of pain in the rabies immunoglobulin chest. After taking history and performing E. Preventive immunization with anti-rabies physical exam the doctor decided to direct serum the patient to cardiologist for a consultation. What level of medical care is being proposed 32. A 28-year-old woman has made to the patient? an appointment with the family doctor to receive vaccination against influenza. A. Secondary healthcare However, having collected the patient’s B. Emergency healthcare medical history, the doctor claimed this C. Primary healthcare procedure to be absolutely contraindicated D. Tertiare healtcare for this woman. What anamnestic data is the E. Palliative care absolute contraindication to vaccination? 29. A 30-year-old woman made an A. Egg white intolerance appointment with the family doctor for B. Pregnancy at 30 weeks scheduled vaccination of her 2-year-old C. Blood hemoglobin - 109 g/L child. What type of healthcare provides such D. Body temperature - 37.2oC medical services? E. Psoriasis in the remission phase